Which of the following is the least likely characteristic of personal computers?

Auditing CIS Reviewer

Uploaded by

LawrenceValdez

0% found this document useful (0 votes)

1K views

15 pages

Original Title

Auditing_CIS_Reviewer.docx

Copyright

© © All Rights Reserved

Available Formats

DOCX, PDF, TXT or read online from Scribd

Share this document

Did you find this document useful?

Is this content inappropriate?

Report this Document

0% found this document useful (0 votes)

1K views15 pages

Auditing CIS Reviewer

Original Title:

Auditing_CIS_Reviewer.docx

Uploaded by

LawrenceValdez

Full description

AUDITING IN A COMPUTERIZED ENVIRONMENT

1. Which statement is incorrect when auditing in a CIS

environment?

a. A CIS environment exists when a computer of any

type or size is involved in the processing by the entity

of financial information of significance to the audit,

whether that computer is operated by the entity or by

a third party.

b. The auditor should consider how a CIS environment

affects the audit.

c. The use of a computer changes the processing,

storage and communication of financial information

and may affect the accounting and internal control

systems employed by the entity.

d. A CIS environment changes the overall objective and

scope of an audit.

2. Which of the following standards or group of standards is

mostly affected by a computerized information system

environment?

a. General standards

b. Reporting standards

c. Second standard of field work

d. Standards of fieldwork

3. Which of the following is least considered if the auditor has to

determine whether specialized CIS skills are needed in an

audit?

a. The auditor needs to obtain a sufficient

understanding of the accounting and internal control

system affected by the CIS environment.

b. The auditor needs to determine the effect of the CIS

environment on the assessment of overall risk and of

risk at the account balance and class of transactions

level.

c. Design and perform appropriate tests of controls and

substantive procedures.

d. The need of the auditor to make analytical

procedures during the completion stage of audit.

4. It relates to materiality of the financial statement assertions

affected by the computer processing.

a. Threshold

b. Relevance

c. Complexity

d. Significance

5. Which of the following least likely indicates a complexity of

computer processing?

a. Transactions are exchanged electronically with other

organizations without manual review of their

propriety.

b. The volume of the transactions is such that users

would find it difficult to identify and correct errors in

processing.

c. The computer automatically generates material

transactions or entries directly to another

applications.

d. The system generates a daily exception report

6. The nature of the risks and the internal characteristics in CIS

environment that the auditors are mostly concerned include the

following except:

a. Lack of segregation of functions.

b. Lack of transaction trails.

c. Dependence of other control over computer

processing.

d. Cost-benefit ratio.

7. Which of the following is least likely a risk characteristic

associated with CIS environment?

a. Errors embedded in an application’s program logic

maybe difficult to manually detect on a timely basis.

b. Many control procedures that would ordinarily be

performed by separate individuals in manual system

maybe concentrated in CIS.

c. The potential unauthorized access to data or to alter

them without visible evidence maybe greater.

d. Initiation of changes in the master file is exclusively

handled by respective users.

8. Which of the following significance and complexity of the CIS

activities should an auditor least understand?

a. The organizational structure of the client’s CIS

activities.

b. Lack of transaction trails.

c. The significance and complexity of computer

processing in each significant accounting application.

d. The use of software packages instead of customized

software.

9. Which statement is correct regarding personal computer

systems?

a. Personal computers or PCs are economical yet

powerful self-contained general purpose computers

consisting typically of a central processing unit

(CPU), memory, monitor, disk drives, printer cables

and modems.

b. Programs and data are stored only on non-

removable storage media.

c. Personal computers cannot be used to process

accounting transactions and produce reports that are

essential to the preparation of financial statements.

d. Generally, CIS environments in which personal

computers are used are the same with other CIS

environments.

10. A personal computer can be used in various configurations,

including

a. A stand-alone workstation operated by a single user

or a number of users at different times.

b. A workstation which is part of a local area network of

personal computers.

c. A workstation connected to a server.

d. All of the above.

11. Which statement is incorrect regarding personal computer

configurations?

a. The stand-alone workstation can be operated by a

single user or a number of users at different times

accessing the same or different programs.

b. A stand-alone workstation may be referred to as a

distributed system.

c. A local area network is an arrangement where two

or more personal computers are linked together

through the use of special software and

communication lines.

d. Personal computers can be linked to servers and

used as part of such systems, for example, as an

intelligent on-line workstation or as part of a

distributed accounting system.

12. Which of the following is the least likely characteristic of

personal computers?

a. They are small enough to be transportable.

b. They are relatively expensive.

c. They can be placed in operation quickly.

d. The operating system software is less

comprehensive than that found in larger computer

environments.

13. Which of the following is an inherent characteristic of software

package?

a. They are typically used without modifications of the

programs.

b. The programs are tailored-made according to the

specific needs of the user.

c. They are developed by software manufacturer

according to a particular user’s specifications.

d. It takes a longer time of implementation.

14. Which of the following is not normally a removable storage

media?

a. Compact disk

b. Tapes

c. Diskettes

d. Hard disk

15. It is a computer program (a block of executable code) that

attaches itself to a legitimate program or data file and uses

itself as a transport mechanism to reproduce itself without the

knowledge of the user.

a. Virus

b. System management program

c. Utility program

d. Encryption

16. Which statement is incorrect regarding internal control in

personal computer environment?

a. Generally, the CIS environment in which personal

computers are used is less structured than a

centrally-controlled CIS environment.

b. Controls over the system development process and

operations may not be viewed by the developer, the

user or management as being as important or cost-

effective.

c. In almost all commercially available operating

systems, the built-in security provided has gradually

increased over the years.

d. In a typical personal computer environment, the

distinction between general CIS controls and CIS

application controls is easily ascertained.

17. Personal computers are susceptible to theft, physical damage,

unauthorized access or misuse of equipment. Which of the

following is least likely a physical security to restrict access to

personal computers when not in use?

a. Using door locks or other security protection during

non-business hours.

b. Fastening the personal computer to a table using

security cables.

c. Locking the personal computer in a protective

cabinet or shell.

d. Using anti-virus software programs.

18. Which of the following is not likely a control over removable

storage media to prevent misplacement, alteration without

authorization or destruction?

a. Using cryptography, which is the process of

transforming programs and information into an

unintelligible form.

b. Placing responsibility for such media under

personnel whose responsibilities include duties of

software custodians or librarians.

c. Using a program and data file check-in and check-

out system and locking the designated storage

locations.

d. Keeping current copies of diskettes, compact disks

or back-up tapes and hard disks in a fireproof

container, either on-site, off-site or both.

19. 19. Which of the following least likely protects critical and

sensitive information from unauthorized access in a personal

computer environment?

a. Using secret file names and hiding the files.

b. Keeping of back up copies offsite.

c. Employing passwords.

d. Segregating data into files organized under separate

file directories.

20. It refers to plans made by the entity to obtain access to

comparable hardware, software and data in the event of their

failure, loss or destruction.

a. Back-up

b. Encryption

c. Anti-virus

d. Wide Area Network (WAN)

21. The effect of personal computers on the accounting system

and the associated risks will least likely depend on

a. The extent to which the personal computer is being

used to process accounting applications.

b. The type and significance of financial transactions

being processed.

c. The nature of files and programs utilized in the

applications.

d. The cost of personal computers.

22. The auditor may often assume that control risk is high in

personal computer systems since, it may not be practicable or

cost-effective for management to implement sufficient controls

to reduce the risks of undetected errors to a minimum level.

This least likely entail

a. More physical examination and confirmation of

assets.

b. More analytical procedures than tests of details.

c. Larger sample sizes.

d. Greater use of computer-assisted audit techniques,

where appropriate.

23. Computer systems that enable users to access data and

programs directly through workstations are referred to as

a. On-line computer systems

b. Personal computer systems

c. Database management systems (DBMS)

d. Database systems

24. On-line systems allow users to initiate various functions

directly. Such functions include:

I. Entering transactions

II. Requesting reports

III. Making inquiries

IV. Updating master files

a. I, II, III and IV

b. I and II

c. I, II and III

d. I and IV

25. Many different types of workstations may be used in on-line

computer systems. The functions performed by these

workstations least likely depend on their

a. Logic

b. Transmission

c. Storage

d. Cost

26. Types of workstations include General Purpose Terminals and

Special Purpose Terminals.Special Purpose Terminals

include

a. Basic keyboard and monitor

b. Point of sale devices

c. Intelligent terminal

d. Personal computers

27. Special Purpose Terminal used to initiate, validate, record,

transmit and complete various banking transactions

a. Automated teller machines

b. Intelligent terminal

c. Point of sale devices

d. Personal computers

28. Which statement is incorrect regarding workstations?

a. Workstations may be located either locally or at

remote sites.

b. Local workstations are connected directly to the

computer through cables.

c. Remote workstations require the use of

telecommunications to link them to the computer.

d. Workstations cannot be used by many users, for

different purposes, in different locations all at the

same time.

29. On-line computer systems may be classified according to

a. How information is entered into the system.

b. How it is processed.

c. When the results are available to the user.

d. All of the above.

30. In an on-line/real time processing system

a. Individual transactions are entered at workstations,

validated and used to update related computer files

immediately.

b. Individual transactions are entered at a workstation,

subjected to certain validation checks and added to a

transaction file that contains other transactions

entered during the period.

c. Individual transactions immediately update a memo

file containing information which has been extracted

from the most recent version of the master file.

d. The master files are updated by other systems.

31. It combines on-line/real time processing and on-line/batch

processing.

a. On-Line/Memo Update (and Subsequent Processing)

b. On-Line Downloading/Uploading Processing

c. On-Line/Inquiry

d. On-Line/Combined Processing

32. It is a communication system that enables computer users to

share computer equipment, application software, data and

voice and video transmissions.

a. Network

b. File server

c. Host

d. Client

33. A type of network that multiple buildings are close enough to

create a campus, but the space between the buildings is not

under the control of the company is

a. Local Area Network (LAN)

b. Metropolitan Area Network (MAN)

c. Wide Area Network (WAN)

d. World Wide Web (WWW)

34. Which of the following is least likely a characteristic of Wide

Area Network (WAN)?

a. Created to connect two or more geographically

separated LANs.

b. Typically involves one or more long-distance

providers, such as a telephone company to provide

the connections.

c. WAN connections tend to be faster than LAN.

d. Usually more expensive than LAN.

35. Gateway is

a. A hardware and software solution that enables

communications between two dissimilar networking

systems or protocols.

b. A device that forwards frames based on destination

addresses.

c. A device that connects and passes packets between

two network segments that use the same

communication protocol.

d. A device that regenerates and retransmits the signal

on a network.

36. A device that works to control the flow of data between two or

more network segments

a. Bridge

b. Router

c. Repeater

d. Switch

37. The undesirable characteristics of on-line computer systems

least likely include

a. Data are usually subjected to immediate validation

checks.

b. Unlimited access of users to all of the functions in a

particular application.

c. Possible lack of visible transaction trail.

d. Potential programmer access to the system.

38. Certain general CIS controls that are particularly important to

on-line processing least likely include

a. Access controls.

b. System development and maintenance controls.

c. Edit, reasonableness and other validation tests.

d. Use of anti-virus software program.

39. Certain CIS application controls that are particularly important

to on-line processing least likely include

a. Pre-processing authorization.

b. Transaction logs.

c. Cut-off procedures.

d. Balancing.

40. Risk of fraud or error in on-line systems may be reduced in the

following circumstances, except

a. If on-line data entry is performed at or near the point

where transactions originate, there is less risk that

the transactions will not be recorded.

b. If invalid transactions are corrected and re-entered

immediately, there is less risk that such transactions

will not be corrected and re-submitted on a timely

basis.

c. If data entry is performed on-line by individuals who

understand the nature of the transactions involved,

the data entry process may be less prone to errors

than when it is performed by individuals unfamiliar

with the nature of the transactions.

d. On-line access to data and programs through

telecommunications may provide greater opportunity

for access to data and programs by unauthorized

persons.

41. Risk of fraud or error in on-line computer systems may be

increased for the following reasons, except

a. If workstations are located throughout the entity, the

opportunity for unauthorized use of a workstation and

the entry of unauthorized transactions may increase.

b. Workstations may provide the opportunity for

unauthorized uses such as modification of previously

entered transactions or balances.

c. If on-line processing is interrupted for any reason,

for example, due to faulty telecommunications, there

may be a greater chance that transactions or files

may be lost and that the recovery may not be

accurate and complete.

d. If transactions are processed immediately on-line,

there is less risk that they will be processed in the

wrong accounting period.

42. 42. The following matters are of particular importance to the

auditor in an on-line computer system, except

a. Authorization, completeness and accuracy of on-line

transactions.

b. Integrity of records and processing, due to on-line

access to the system by many users and

programmers.

c. Changes in the performance of audit procedures

including the use of CAAT's.

d. Cost-benefit ratio of installing on-line computer

system.

43. A collection of data that is shared and used by a number of

different users for different purposes.

a. Database

b. Information file

c. Master file

d. Transaction file

44. Which of the following is least likely a characteristic of a

database system?

a. Individual applications share the data in the database

for different purposes.

b. Separate data files are maintained for each

application and similar data used by several

applications may be repeated on several different

files.

c. A software facility is required to keep track of the

location of the data in the database.

d. Coordination is usually performed by a group of

individuals whose responsibility is typically referred to

as "database administration."

45. Database administration tasks typically include

I. Defining the database structure.

II. Maintaining data integrity, security and

completeness.

III. Coordinating computer operations related to the

database.

IV. Monitoring system performance.

V. Providing administrative support.

a. All of the above

b. All except I

c. II and V only

d. II, III and V only

46. Due to data sharing, data independence and other

characteristics of database systems

a. General CIS controls normally have a greater

influence than CIS application controls on database

systems.

b. CIS application controls normally have a greater

influence than general CIS controls on database

systems.

c. General CIS controls normally have an equal

influence with CIS application controls on database

systems.

d. CIS application controls normally have no influence

on database systems.

47. Which statement is incorrect regarding the general CIS controls

of particular importance in a database environment?

a. Since data are shared by many users, control may

be enhanced when a standard approach is used for

developing each new application program and for

application program modification.

b. Several data owners should be assigned

responsibility for defining access and security rules,

such as who can use the data (access) and what

functions they can perform (security).

c. User access to the database can be restricted

through the use of passwords.

d. Responsibilities for performing the various activities

required to design, implement and operate a

database are divided among technical, design,

administrative and user personnel.

48. These require a database administrator to assign security

attributes to data that cannot be changed by database users.

a. Discretionary access controls

b. Name-dependent restrictions

c. Mandatory access controls

d. Content-dependent restrictions.

49. A discretionary access control wherein users are permitted or

denied access to data resource depending on the time series

of accesses to and actions they have undertaken on data

resources.

a. Name-dependent restrictions

b. Context-dependent restriction

c. Content-dependent restriction

d. History-dependent restriction

50. The effect of a database system on the accounting system and

the associated risks will least likely depend on:

a. The extent to which databases are being used by

accounting applications.

b. The type and significance of financial transactions

being processed.

c. The nature of the database, the DBMS, the database

administration tasks and the applications.

d. The CIS application controls.

51. Audit procedures in a database environment will be affected

principally by

a. The extent to which the data in the database are

used by the accounting system.

b. The type and significance of financial transactions

being processed.

c. The nature of the database, the DBMS, the database

administration tasks and the applications.

d. The general CIS controls which are particularly

important in a database environment.

52. Which statement is incorrect regarding the characteristics of a

CIS organizational structure?

a. Certain data processing personnel may be the only

ones with a detailed knowledge of the

interrelationship between the source of data, how it is

processed and the distribution and use of the output.

b. Many conventional controls based on adequate

segregation of incompatible functions may not exist,

or in the absence of access and other controls, may

be less effective.

c. Transaction and master file data are often

concentrated, usually in machine-readable form,

either in one computer installation located centrally or

in a number of installations distributed throughout an

entity.

d. Systems employing CIS methods do not include

manual operations since the number of persons

involved in the processing of financial information is

significantly reduced.

53. System characteristics that may result from the nature of CIS

processing include, except

a. Absence of input documents.

b. Lack of visible transaction trail.

c. Lack of visible output.

d. Difficulty of access to data and computer programs.

54. The development of CIS will generally result in design and

procedural characteristics that are different from those found in

manual systems. These different design and procedural

aspectsof CIS include, except:

a. Consistency of performance.

b. Programmed control procedures.

c. Vulnerability of data and program storage media

d. Multiple transaction update of multiple computer files

or databases.

55. Which statement is incorrect regarding internal controls in a

CIS environment?

a. Manual and computer control procedures comprise

the overall controls affecting the CIS environment

(general CIS controls) and the specific controls over

the accounting applications (CIS application

controls).

b. The purpose of general CIS controls is to establish a

framework of overall control over the CIS activities

and to provide a reasonable level of assurance that

the overall objectives of internal control are achieved.

c. The purpose of CIS application controls is to

establish specific control procedures over the

application systems in order to provide reasonable

assurance that all transactions are authorized and

recorded, and are processed completely, accurately

and on a timely basis.

d. The internal controls over computer processing,

which help to achieve the overall objectives of

internal control, include only the procedures

designed into computer programs.

56. General CIS controls may include, except:

a. Organization and management controls.

b. Delivery and support controls.

c. Development and maintenance controls.

d. Controls over computer data files.

57. 57. CIS application controls include, except

a. Controls over input.

b. Controls over processing and computer data files.

c. Controls over output.

d. Monitoring controls.

58. Which statement is incorrect regarding the review of general

CIS controls and CIS application controls?

a. The auditor should consider how these general CIS

controls affect the CIS applications significant to the

audit.

b. General CIS controls that relate to some or all

applications are typically interdependent controls in

that their operation is often essential to the

effectiveness of CIS application controls.

c. Control over input, processing, data files and output

may be carried out by CIS personnel, by users of the

system, by a separate control group, or may be

programmed into application software.

d. It may be more efficient to review the design of the

application controls before reviewing the general

controls.

59. Which statement is incorrect regarding the evaluation of

general CIS controls and CIS application controls?

a. The general CIS controls may have a pervasive

effect on the processing of transactions in application

systems.

b. If general CIS controls are not effective, there may

be a risk that misstatements might occur and go

undetected in the application systems.

c. Manual procedures exercised by users may provide

effective control at the application level.

d. Weaknesses in general CIS controls cannot preclude

testing certain CIS application controls.

60. The applications of auditing procedures using the computer as

an audit tool refer to

a. Integrated test facility

b. Auditing through the computer

c. Data-based management system

d. Computer assisted audit techniques

61. Which statement is incorrect regarding CAATs?

a. CAATs are often an efficient means of testing a large

number of transactions or controls over large

populations.

b. To ensure appropriate control procedures, the

presence of the auditor is not necessarily required at

the computer facility during the running of a CAAT.

c. The general principles outlined in PAPS 1009 apply

in small entity IT environments.

d. Where smaller volumes of data are processed, the

use of CAATs is more cost effective.

62. Consists of generalized computer programs designed to

perform common audit tasks or standardized data processing

functions.

a. Package or generalized audit software

b. Utility programs

c. Customized or purpose-written programs

d. System management programs

63. Audit automation least likely include

a. Expert systems.

b. Tools to evaluate a client’s risk management

procedures.

c. Manual working papers.

d. Corporate and financial modeling programs for use

as predictive audit tests.

64. An internal auditor noted the following points when conducting

a preliminary survey in connection with the audit of an EDP

department. Which of the following would be considered a

safeguard in the control system on which the auditor might

rely?

a. Programmers and computer operators correct daily

processing problems as they arise.

b. The control group works with user organizations to

correct rejected input.

c. New systems are documented as soon as possible

after they begin processing live data.

d. The average tenure of employees working in the

EDP department is ten months.

65. An on-line access control that checks whether the user’s code

number is authorized to initiate a specific type of transaction or

inquiry is referred to as

a. Password

b. Compatibility test

c. Limit check

d. Reasonableness test

66. A control procedure that could be used in an on-line system to

provide an immediate check on whether an account number

has been entered on a terminal accurately is a

a. Compatibility test

b. Record count

c. Hash total

d. Self-checking digit

67. A control designed to catch errors at the point of data entry is

a. Batch total

b. Self-checking digit

c. Record count

d. Checkpoints

68. Program documentation is a control designed primarily to

ensure that

a. Programmers have access to the tape library or

information on disk files.

b. Programs do not make mathematical errors.

c. Programs are kept up to date and perform as

intended.

d. Data have been entered and processed.

69. Some of the more important controls that relate to automated

accounting information systems are validity checks, limit

checks, field checks, and sign tests. These are classified as

a. Control total validation routines

b. Output controls

c. Hash totaling

d. Input validation routines

70. Most of today’s computer systems have hardware controls that

are built in by the computer manufacturer. Common hardware

controls are

a. Duplicate circuitry, echo check, and internal header

labels

b. Tape file protection, cryptographic protection, and

limit checks

c. Duplicate circuitry, echo check, and dual reading

d. Duplicate circuitry, echo check, tape file protection,

and internal header labels

71. Computer manufacturers are now installing software programs

permanently inside the computer as part of its main memory to

provide protection from erasure or loss if there is interrupted

electrical power. This concept is known as

a. File integrity

b. Random access memory (RAM)

c. Software control

d. Firmware

72. Which one of the following represents a lack of internal control

in a computer-based information system?

a. The design and implementation is performed in

accordance with management’s specific

authorization.

b. Any and all changes in application programs have

the authorization and approval of management.

c. Provisions exist to protect data files from

unauthorized access, modification, or destruction.

d. Both computer operators and programmers have

unlimited access to the programs and data files.

73. In an automated payroll processing environment, a department

manager substituted the time card for a terminated employee

with a time card for a fictitious employee. The fictitious

employee had the same pay rate and hours worked as the

terminated employee. The best control technique to detect this

action using employee identification numbers would be a

a. Batch total

b. Hash total

c. Record count

d. Subsequent check

74. An employee in the receiving department keyed in a shipment

from a remote terminal and inadvertently omitted the purchase

order number. The best systems control to detect this error

would be

a. Batch total

b. Sequence check

c. Completeness test

d. Reasonableness test

75. The reporting of accounting information plays a central role in

the regulation of business operations. Preventive controls are

an integral part of virtually all accounting processing systems,

and much of the information generated by the accounting

system is used for preventive control purposes. Which one of

the following is not an essential element of a sound preventive

control system?

a. Separation of responsibilities for the recording,

custodial, and authorization functions.

b. Sound personnel policies.

c. Documentation of policies and procedures.

d. Implementation of state-of-the-art software and

hardware.

76. The most critical aspect regarding separation of duties within

information systems is between

a. Project leaders and programmers

b. Programmers and systems analysts

c. Programmers and computer operators

d. Data control and file librarians

77. Whether or not a real time program contains adequate controls

is most effectively determined by the use of

a. Audit software

b. A tracing routine

c. An integrated test facility

d. A traditional test deck

78. Compatibility tests are sometimes employed to determine

whether an acceptable user is allowed to proceed. In order to

perform compatibility tests, the system must maintain an

access control matrix. The one item that is not part of an

access control matrix is a

a. List of all authorized user code numbers and

passwords.

b. List of all files maintained on the system.

c. Record of the type of access to which each user is

entitled.

d. Limit on the number of transaction inquiries that can

be made by each user in a specified time period.

79. Which one of the following input validation routines is not likely

to be appropriate in a real time operation?

a. Field check

b. Sequence check

c. Sign check

d. Redundant data check

80. Which of the following controls is a processing control designed

to ensure the reliability and accuracy of data processing?

Limit test Validity check test

a. Yes Yes

b. No No

c. No Yes

d. Yes No

81. Which of the following characteristics distinguishes computer

processing from manual processing?

a. Computer processing virtually eliminates the

occurrence of computational error normally

associated with manual processing.

b. Errors or irregularities in computer processing will be

detected soon after their occurrences.

c. The potential for systematic error is ordinarily greater

in manual processing than in computerized

processing.

d. Most computer systems are designed so that

transaction trails useful for audit do not exist.

82. Which of the following most likely represents a significant

deficiency in the internal control structure?

a. The systems analyst review applications of data

processing and maintains systems documentation.

b. The systems programmer designs systems for

computerized applications and maintains output

controls.

c. The control clerk establishes control over data

received by the EDP department and reconciles

control totals after processing

d. The accounts payable clerk prepares data for

computer processing and enters the data into the

computer.

83. Which of the following activities would most likely be performed

in the EDP Department?

a. Initiation of changes to master records.

b. Conversion of information to machine-readable form.

c. Correction of transactional errors.

d. Initiation of changes to existing applications.

84. For control purposes, which of the following should be

organizationally segregated from the computer operations

function?

a. Data conversion

b. Systems development

c. Surveillance of CRT messages

d. Minor maintenance according to a schedule

85. Which of the following is not a major reason for maintaining an

audit trail for a computer system?

a. Deterrent to irregularities

b. Analytical procedures

c. Monitoring purposes

d. Query answering

86. In an automated payroll system, all employees in the finishing

department were paid the rate of P75 per hour when the

authorized rate was P70 per hour. Which of the following

controls would have been most effective in preventing such an

error?

a. Access controls which would restrict the personnel

department’s access to the payroll master file data.

b. A review of all authorized pay rate changes by the

personnel department.

c. The use of batch control totals by department.

d. A limit test that compares the pay rates per

department with the maximum rate for all employees.

87. Which of the following errors would be detected by batch

controls?

a. A fictitious employee as added to the processing of

the weekly time cards by the computer operator.

b. An employee who worked only 5 hours in the week

was paid for 50 hours.

c. The time card for one employee was not processed

because it was lost in transit between the payroll

department and the data entry function.

d. All of the above.

88. The use of a header label in conjunction with magnetic tape is

most likely to prevent errors by the

a. Computer operator

b. Computer programmer

c. Keypunch operator

d. Maintenance technician

89. For the accounting system of ACME Company, the amounts of

cash disbursements entered into an EDP terminal are

transmitted to the computer that immediately transmits the

amounts back to the terminal for display on the terminal screen.

This display enables the operator to

a. Establish the validity of the account number

b. Verify the amount was entered accurately

c. Verify the authorization of the disbursements

d. Prevent the overpayment of the account

90. When EDP programs or files can be accessed from terminals,

users should be required to enter a(an)

a. Parity check

b. Self-diagnostic test

c. Personal identification code

d. Echo check

91. The possibility of erasing a large amount of information stored

on magnetic tape most likely would be reduced by the use of

a. File protection ring

b. Completeness tests

c. Check digits

d. Conversion verification

92. Which of the following controls most likely would assure that an

entity can reconstruct its financial records?

a. Hardware controls are built into the computer by the

computer manufacturer.

b. Backup diskettes or tapes of files are stored away

from originals.

c. Personnel who are independent of data input

perform parallel simulations.

d. System flowcharts provide accurate descriptions of

input and output operations.

93. Mill Co. uses a batch processing method to process its sales

transactions. Data on Mill’s sales transaction tape are

electronically sorted by customer number and are subject to

programmed edit checks in preparing its invoices, sales

journals, and updated customer account balances. One of the

direct outputs of the creation of this tape most likely would be a

a. Report showing exceptions and control totals.

b. Printout of the updated inventory records.

c. Report showing overdue accounts receivable.

d. Printout of the sales price master file.

94. Using microcomputers in auditing may affect the methods used

to review the work of staff assistants because

a. The audit field work standards for supervision may

differ.

b. Documenting the supervisory review may require

assistance of consulting services personnel.

c. Supervisory personnel may not have an

understanding of the capabilities and limitations of

microcomputers.

d. Working paper documentation may not contain

readily observable details of calculations.

95. An auditor anticipates assessing control risk at a low level in a

computerized environment. Under these circumstances, on

which of the following procedures would the auditor initially

focus?

a. Programmed control procedures

b. Output control procedures

c. Application control procedures

d. General control procedures

96. After the preliminary phase of the review of a client’s EDP

controls, an auditor may decide not to perform tests of controls

(compliance tests) related to the control procedures within the

EDP portion of the client’s internal control structure. Which of

the following would not be a valid reason for choosing to omit

such tests?

a. The controls duplicate operative controls existing

elsewhere in the structure.

b. There appear to be major weaknesses that would

preclude reliance on the stated procedure.

c. The time and costs of testing exceed the time and

costs in substantive testing if the tests of controls

show the controls to be operative.

d. The controls appear adequate.

97. Which of the following client electronic data processing (EDP)

systems generally can be audited without examining or directly

testing the EDP computer programs of the system?

a. A system that performs relatively uncomplicated

processes and produces detailed output.

b. A system that affects a number of essential master

files and produces a limited output.

c. A system that updates a few essential master files

and produces no printed output other than final

balances.

d. A system that performs relatively complicated

processing and produces very little detailed output.

98. Computer systems are typically supported by a variety of utility

software packages that are important to an auditor because

they

a. May enable unauthorized changes to data files if not

properly controlled.

b. Are very versatile programs that can be used on

hardware of many manufacturers.

c. May be significant components of a client’s

application programs.

d. Are written specifically to enable auditors to extract

and sort data.

99. To obtain evidence that online access controls are properly

functioning, an auditor most likely would

a. Create checkpoints at periodic intervals after live

data processing to test for unauthorized use of the

system.

b. Examine the transaction log to discover whether any

transactions were lost or entered twice due to a

system malfunction

c. Enter invalid identification numbers or passwords to

ascertain whether the system rejects them.

d. Vouch a random sample of processed transactions to

assure proper authorization

100. Which of the following statements most likely represents a

disadvantage for an entity that keeps microcomputer-prepared

data files rather than manually prepared files?

a. Attention is focused on the accuracy of the

programming process rather than errors in individual

transactions.

b. It is usually easier for unauthorized persons to

access and alter the files.

c. Random error associated with processing similar

transactions in different ways is usually greater.

d. It is usually more difficult to compare recorded

accountability with physical count of assets.

101. An auditor would least likely use computer software to

a. Access client data files

b. Assess EDP controls

c. Prepare spreadsheets

d. Construct parallel simulations

102. A primary advantage of using generalized audit software

packages to audit the financial statements of a client that uses

an EDP system is that the auditor may

a. Consider increasing the use of substantive tests of

transactions in place of analytical procedures.

b. Substantiate the accuracy of data through self-

checking digits and hash totals.

c. Reduce the level of required tests of controls to a

relatively small amount.

d. Access information stored on computer files while

having a limited understanding of the client’s

hardware and software features.

103. Auditors often make use of computer programs that perform

routine processing functions such as sorting and merging.

These programs are made available by electronic data

processing companies and others and are specifically referred

to as

a. Compiler programs

b. Utility programs

c. Supervisory programs

d. User programs

104. Smith Corporation has numerous customers. A customer file is

kept on disk storage. Each customer file contains name,

address, credit limit, and account balance. The auditor wishes

to test this file to determine whether the credit limits are being

exceeded. The best procedure for the auditor to follow would

be to

a. Develop test data that would cause some account

balances to exceed the credit limit and determine if

the system properly detects such situations.

b. Develop a program to compare credit limits with

account balances and print out the details of any

account with a balance exceeding its credit limit.

c. Request a printout of all account balances so they

can be manually checked against the credit limits.

d. Request a printout of a sample of account balances

so they can be individually checked against the credit

limits.

105. The use of generalized audit software package

a. Relieves an auditor of the typical tasks of

investigating exceptions, verifying sources of

information, and evaluating reports.

b. Is a major aid in retrieving information from

computerized files.

c. Overcomes the need for an auditor to learn much

about computers.

d. Is a form of auditing around the computer.

106. An auditor used test data to verify the existence of controls in a

certain computer program. Even though the program

performed well on the test, the auditor may still have a concern

that

a. The program tested is the same one used in the

regular production runs.

b. Generalized audit software may have been a better

tool to use.

c. Data entry procedures may change and render the

test useless.

d. The test data will not be relevant in subsequent audit

periods.

107. An auditor most likely would introduce test data into a

computerized payroll system to test internal controls related to

the

a. Existence of unclaimed payroll checks held by

supervisors.

b. Early cashing of payroll checks by employees.

c. Discovery of invalid employee I.D. numbers.

d. Proper approval of overtime by supervisors.

108. When an auditor tests a computerized accounting system,

which of the following is true of the test data approach?

a. Test data must consist of all possible valid and

invalid conditions.

b. The program tested is different from the program

used throughout the year by the client.

c. Several transactions of each type must be tested.

d. Test data are processed by the client’s computer

programs under the auditor’s control.

109. Which of the following statements is not true to the test data

approach when testing a computerized accounting system?

a. The test need consist of only those valid and invalid

conditions which interest the auditor

b. Only one transaction of each type need be tested.

c. The test data must consist of all possible valid and

invalid conditions.

d. Test data are processed by the client’s computer

programs under the auditor’s control.

110. Which of the following is not among the errors that an auditor

might include in the test data when auditing a client’s EDP

system?

a. Numeric characters in alphanumeric fields.

b. Authorized code.

c. Differences in description of units of measure.

d. Illogical entries in fields whose logic is tested by

programmed consistency checks.

111. An auditor who is testing EDP controls in a payroll system

would most likely use test data that contain conditions such as

a. Deductions not authorized by employees.

b. Overtime not approved by supervisors.

c. Time tickets with invalid job numbers.

d. Payroll checks with unauthorized signatures.

112. Auditing by testing the input and output of an EDP system

instead of the computer program itself will

a. Not detect program errors which do not show up in

the output sampled.

b. Detect all program errors, regardless of the nature of

the output.

c. Provide the auditor with the same type of evidence.

d. Not provide the auditor with confidence in the results

of the auditing procedures.

113. Which of the following computer-assisted auditing techniques

allows fictitious and real transactions to be processed together

without client operating personnel being aware of the testing

process?

a. Integrated test facility

b. Parallel simulation

c. Input controls matrix

d. Data entry monitor

114. Which of the following methods of testing application controls

utilizes a generalized audit software package prepared by the

auditors?

a. Parallel simulation

b. Test data approach

c. Integrated testing facility approach

d. Exception report tests

115. Misstatements in a batch computer system caused by incorrect

programs or data may not be detected immediately because

a. Errors in some transactions may cause rejection of

other transactions in the batch.

b. The identification of errors in input data typically is

not part of the program.

c. There are time delays in processing transactions in a

batch system.

d. The processing of transactions in a batch system is

not uniform.

116. Which of the following is not a characteristic of a batch

processed computer system?

a. The collection of like transactions which are sorted

and processed sequentially against a master file.

b. Keypunching of transactions, followed by machine

processing.

c. The production of numerous printouts.

d. The posting of a transaction, as it occurs, to several

files, without immediate printouts.

117. Where disk files are used, the grandfather-father-son updating

backup concept is relatively

118. difficult to implement because the

a. Location of information points on disks is an

extremely time consuming task.

b. Magnetic fields and other environmental factors

cause off-site storage to be impractical.

c. Information must be dumped in the form of hard copy

if it is to be reviewed before used in

d. Process of updating old records is destructive.

119. An auditor would most likely be concerned with which of the

following controls in a distributed data processing system?

a. Hardware controls

b. Access controls

c. Systems documentation controls

d. Disaster recovery controls

120. If a control total were computed on each of the following data

items, which would best be identified as a hash total for a

payroll EDP application?

a. Total debits and total credits

b. Department numbers

c. Net pay

d. Hours worked

121. Which of the following is a computer test made to ascertain

whether a given characteristic belongs to the group?

a. Parity check

b. Echo check

c. Validity check

d. Limit check

122. A control feature in an electronic data processing system

requires the central processing unit (CPU) to send signals to

the printer to activate the print mechanism for each character.

The print mechanism, just prior to printing, sends a signal back

to the CPU verifying that the proper print position has been

activated. This type of hardware control is referred to as

a. Echo check

b. Signal control

c. Validity control

d. Check digit control

123. Which of the following is an example of a check digit?

a. An agreement of the total number of employees to

the total number of checks printed by the computer.

b. An algebraically determined number produced by the

other digits of the employee number

c. A logic test that ensures all employee numbers are

nine digits.

d. A limit check that an employee’s hours do not exceed

50 hours per work week.

124. In a computerized system, procedure or problem-oriented

language is converted to machine language through a(an)

a. Interpreter

b. Verifier

c. Compiler

d. Converter

125. A customer erroneously ordered Item No. 86321 rather than

item No. 83621. When this order is processed, the vendor’s

EDP department would identify the error with what type of

control?

a. Key verifying

b. Batch total

c. Self-checking digit

d. Item inspection

126. The computer process whereby data processing is performed

concurrently with a particular activity and the results are

available soon enough to influence the course of action being

taken or the decision being made is called:

a. Random access sampling

b. On-line, real-time system

c. Integrated data processing

d. Batch processing system

127. Internal control is ineffective when computer department

personnel

a. Participate in computer software acquisition

decisions.

b. Design documentation for computerized systems.

c. Originate changes in master file.

d. Provide physical security for program files.

128. Test data, integrated test data and parallel simulation each

require an auditor to prepare data and computer programs.

CPAs who lack either the technical expertise or time to prepare

programs should request from the manufacturers or EDP

consultants for

a. The program Code

b. Generalized audit software

c. Flowchart checks

d. Application controls

129. Which of the following best describes a fundamental control

weakness often associated with electronic data processing

system?

a. EDP equipment is more subject to system error than

manual processing is subject to human error.

b. Monitoring is not an adequate substitute for the use

of test data.

c. EDP equipment processes and records similar

transactions in a similar manner.

d. Functions that would normally be separated in a

manual system are combined in the EDP system like

the function of programmers and operators.

130. Which of the following tasks could not be performed when

using a generalized audit software package?

a. Selecting inventory items for observations.

b. Physical count of inventories.

c. Comparison of inventory test counts with perpetual

records.

d. Summarizing inventory turnover statistics for

obsolescence analysis.

131. All of the following are “auditing through the computer”

techniques except

a. Reviewing source code

b. Automated tracking and mapping

c. Test-decking

d. Integrated test facility

132. The output of a parallel simulation should always be

a. Printed on a report.

b. Compared with actual results manually.

c. Compared with actual results using a comparison

program.

d. Reconciled to actual processing output.

133. Generalized audit software is a computer-assisted audit

technique. It is one of the widely used technique for auditing

computer application systems. Generalized audit software is

most often used to

a. Verify computer processing.

b. Process data fields under the control of the operation

manager.

c. Independently analyze data files.

d. Both a and b.

134. From an audit viewpoint, which of the following represents a

potential disadvantage associated with the widespread use of

microcomputers?

a. Their portability.

b. Their ease of access by novice users.

c. Their easily developed programs using spreadsheets

which do not have to be documented.

d. All of the above.

135. Which of the following functions would have the least effect on

an audit if it was not properly segregated?

a. The systems analyst and the programmer functions.

b. The computer operator and programmer functions.

c. The computer operator and the user functions.

d. The applications programmer and the systems

programmer.

136. To obtain evidence that user identification and password

control procedures are functioning as designed, an auditor

would most likely

a. Attempt to sign on to the system using invalid user

identifications and passwords.

b. Write a computer program that simulates the logic of

the client’s access control software.

c. Extract a random sample of processed transactions

and ensure that the transactions were appropriately

authorized. Examine statements signed by

employees stating that they have not divulged their

user identifications and passwords to any other

person.

137. In considering a client's internal control structure in a computer

environment, the auditor will encounter general controls and

application controls. Which of the following is an application

control?

a. Organization charts.

b. Hash total.

c. Systems flowcharts.

d. Control over program changes

138. Auditing by testing the input and output of a computer system--

i.e., auditing "around" the computer--instead of the computer

software itself will

a. Not detect program errors that do not appear in the

output sampled.

b. Detect all program errors, regardless of the nature of

the output.

c. Provide the auditor with the same type of evidence.

d. Not provide the auditor with confidence in the results

of the auditing procedures.

139. Smith Corporation has numerous customers. A customer file is

kept on disk. Each customer file contains the name, address,

credit limit, and account balance. The auditor wishes to test

this file to determine whether credit limits are being exceeded.

The best procedure for the auditor to follow would be to

a. Develop test data that would cause some account

balances to exceed the credit limit and determine if

the system properly detects such situations.

b. Develop a program to compare credit limits with

account balances and print out the details of any

account with a balance exceeding its credit limit.

c. Request a printout of all account balances so they

can be manually checked against the credit limits.

d. Request a printout of a sample of account balances

so they can be individually checked against the credit

limits.

140. Which of the following methods of testing application controls

utilizes software prepared by the auditors and applied to the

client's data?

a. Parallel simulation.

b. Integrated test facility.

c. Test data.

d. Exception report tests.

141. The testdata method is used by auditors to test the

a. Accuracy of input data.

b. Validity of the output.

c. Procedures contained within the program.

d. Normalcy of distribution of test data.

142. Which of the following is true of generalized audit software?

a. They can be used only in auditing on-line computer

systems.

b. They can be used on any computer without

modification.

c. They each have their own characteristics, which the

auditor must carefully consider before using in a

given audit situation.

d. They enable the auditor to perform all manual

compliance test procedures less expensively.

143. Assume that an auditor estimated that 10,000 checks were

issued during the accounting period. If an application control

that performs a limit check for each check request is to be

subjected to the auditor's testdata approach, the sample

should include:

a. Approximately 1,000 test items.

b. A number of test items determined by the auditor to

be sufficient under the circumstances.

c. A number of test items determined by the auditor's

reference to the appropriate sampling tables.

d. One transaction.

144. PC DOS, MS DOS, and AppleDOS are examples of

a. Application software.

b. Generalized audit software.

c. Database management systems.

d. Operating software.

145. Which of the following is not an example of a computer-

assisted audit technique?

a. Integrated test data.

b. Audit modules.

c. Disk operating systems.

d. Audit hooks.

146. Which of the following statements most likely represents a

disadvantage for an entity that maintains computer data files

rather than manual files?

a. It's usually more difficult to detect transposition

errors.

b. Transactions are usually authorized before they are

executed and recorded.

c. It's usually easier for unauthorized persons to access

and alter the files.

d. Random error is more common when similar

transactions are processed in different ways.

147. Which of the following statements best describes a weakness

often associated with computers?

a. Computer equipment is more subject to systems

error than manual processing is subject to human

error.

b. Computer equipment processes and records similar

transactions in a similar manner.

c. Control activities for detecting invalid and unusual

transactions are less effective than manual control

activities.

d. Functions that would normally be separated in a

manual system are combined in a computer system.

148. Accounting functions that are normally considered incompatible

in a manual system are often combined by computer software.

This necessitates an application control that prevents

unapproved

a. Access to the computer library.

b. Revisions to existing software.

c. Usage of software.

d. Testing of modified software.

149. When software or files can be accessed from on-line servers,

users should be required to enter

a. A parity check.

b. A personal identification code.

c. A self-diagnosis test.

d. An echo check.

150. An auditor's consideration of a company's computer control

activities has disclosed the following four circumstances.

Indicate which circumstance constitutes a significant deficiency

in internal control.

a. Computer operators do not have access to the

complete software support documentation.

b. Computer operators are closely supervised by

programmers.

c. Programmers are not authorized to operate

computers.

d. Only one generation of backup files is stored in an

off-premises location.

151. In a computer system, hardware controls are designed to

a. Arrange data in a logical sequence for processing.

b. Correct errors in software.

c. Monitor and detect errors in source documents.

d. Detect and control errors arising from use of

equipment.

152. In the weekly computer run to prepare payroll checks, a check

was printed for an employee who had been terminated the

previous week. Which of the following controls, if properly

utilized, would have been most effective in preventing the error

or ensuring its prompt detection?

a. A control total for hours worked, prepared from time

cards collected by the timekeeping department.

b. Requiring the treasurer's office to account for the

number of the pre-numbered checks issued to the

CBIS department for the processing of the payroll

c. Use of a check digit for employee numbers

d. Use of a header label for the payroll input sheet

153. An auditor is preparing test data for use in the audit of a

computer based accounts receivable application. Which of the

following items would be appropriate to include as an item in

the test data?

a. A transaction record which contains an incorrect

master file control total

b. A master file record which contains an invalid

customer identification number

c. A master file record which contains an incorrect

master file control total

d. A transaction record which contains an invalid

customer identification number.

154. Unauthorized alteration of on-line records can be prevented by

employing:

a. Key verification

b. Computer sequence checks

c. Computer matching

d. Data base access controls

155. In auditing through a computer, the test data method is used by

auditors to test the

a. Accuracy of input data

b. Validity of the output

c. Procedures contained within the program

d. Normalcy of distribution of test data.

156. In the preliminary survey the auditor learns that a department

has several microcomputers. Which of the following is usually

true and should be considered in planning the audit?

a. Microcomputers, though small, are capable of

processing financial information, and physical

security is a control concern

b. Microcomputers are limited to applications such as

worksheet generation and do not present a

significant audit risk

c. Microcomputers are generally under the control of

the data processing department and use the same

control features

d. Microcomputers are too small to contain any built-in

control features. Therefore, other controls must be

relied upon.

157. The primary reason for internal auditing's involvement in the

development of new computer-based sysstems is to:

a. Plan post-implementation reviews

b. Promote adequate controls

c. Train auditors in CBIS techniques

d. Reduce overall audit effort.

158. Which of the following is an advantage of generalized computer

audit packages?

a. They are all written in one identical computer

language

b. They can be used for audits of clients that use

differing CBIS equipment and file formats

c. They have reduced the need for the auditor to study

input controls for CBIS related procedures

d. Their use can be substituted for a relatively large part

of the required control testing

159. Processing simulated file data provides the auditor with

information about the reliability of controls from evidence that

exists in simulated files. One of the techniques involved in this

approach makes use of

a. Controlled reprocessing

b. Program code checking

c. Printout reviews

d. Integrated test facility

160. Which of the following statements most likely represents a

disadvantage for an entity that keeps microcomputer-prepared

data files rather than manually prepared files?

a. It is usually more difficult to detect transposition

errors

b. Transactions are usually authorized before they are

executed and recorded

c. It is usually easier for unauthorized persons to

access and alter the files

d. Random error associated with processing similar

transactions in different ways is usually greater

161. The possibility of losing a large amount of information stored in

computer files most likely would be reduced by the use of

a. Back-up files

b. Check digits

c. Completeness tests

d. Conversion verification

162. An integrated test facility (ITF) would be appropriate when the

auditor needs to

a. Trace a complex logic path through an application

system

b. Verify processing accuracy concurrently with

processing

c. Monitor transactions in an application system

continuously

d. Verify load module integrity for production programs

163. Where computer processing is used in significant accounting

applications, internal accounting control procedures may be

defined by classifying control procedures into two types:

general and

a. Administrative

b. Specific

c. Application

d. Authorization

164. The increased presence of the microcomputer in the workplace

has resulted in an increasing number of persons having access

to the computer. A control that is often used to prevent

unauthorized access to sensitive programs is:

a. Backup copies of the diskettes

b. Passwords for each of the users

c. Disaster-recovery procedures

d. Record counts of the number of input transactions in

a batch being processed

165. Checklists, systems development methodology, and staff hiring

are examples of what type of controls?

a. Detective

b. Preventive

c. Subjective

d. Corrective

166. When an on-line, real-time (OLRT) computer-based processing

system is in use, internal control can be strengthened by

a. Providing for the separation of duties between

keypunching and error listing operations

b. Attaching plastic file protection rings to reels of

magnetic tape before new data can be entered on

the file

c. Making a validity check of an identification number

before a user can obtain access to the computer files

d. Preparing batch totals to provide assurance that file

updates are made for the entire input

167. When auditing "around" the computer, the independent auditor

focuses solely upon the source documents and

a. Test data

b. CBIS processing

c. Control techniques

d. CBIS output

168. One of the features that distinguishes computer processing

from manual processing is

a. Computer processing virtually eliminates the

occurrence of computational error normally

associated with manual processing

b. Errors or fraud in computer processing will be

detected soon after their occurrences

c. The potential for systematic error is ordinarily greater

in manual processing than in computerized

processing

d. Most computer systems are designed so that

transaction trails useful for audit purposes do not

exist

169. Given the increasing use of microcomputers as a means for

accessing data bases, along with on-line real-time processing,

companies face a serious challenge relating to data security.

Which of the following is not an appropriate means for meeting

this challenge?

a. Institute a policy of strict identification and password

controls housed in the computer software that permit

only specified individuals to access the computer

files and perform a given function.

b. Limit terminals to perform only certain transactions.

c. Program software to produce a log of transactions

showing date, time, type of transaction, and operator.

d. Prohibit the networking of microcomputers and do

not permit users to access centralized data bases.

170. What type of computer-based system is characterized by data

that are assembled from more than one location and records

that are updated immediately?

a. Microcomputer system

b. Minicomputer system

c. Batch processing system

d. Online real-time system

171. Company A has recently converted its manual payroll to a

computer-based system. Under the old system, employees

who had resigned or been terminated were occasionally kept

on the payroll and their checks were claimed and cashed by

other employees, in collusion with shop foremen. The

controller is concerned that this practice not be allowed to

continue under the new system. The best control for

preventing this form of "payroll padding" would be to

a. Conduct exit interviews with all employees leaving

the company, regardless of reason.

b. Require foremen to obtain a signed receipt from each

employee claiming a payroll check.

c. Require the human resources department to

authorize all hires and terminations, and to forward a

current computerized list of active employee

numbers to payroll prior to processing. Program the

computer to reject inactive employee numbers.

d. Install time clocks for use by all hourly employees.

172. Compared to a manual system, a CBIS generally

1) Reduces segregation of duties

2) Increases segregation of duties

3) Decreases manual inspection of processing results

4) Increases manual inspection of processing results.

a. 1 and 3

b. 1 and 4

c. 2 and 3

d. 2 and 4

173. One of the major problems in a CBIS is that incompatible

functions may be performed by the same individual. One

compensating control for this is the use of

a. Echo checks

b. A self-checking digit system

c. Computer generated hash totals

d. A computer log

174. Which of the following processing controls would be most

effective in assisting a store manager to ascertain whether the

payroll transaction data were processed in their entirety?

a. Payroll file header record

b. Transaction identification codes

c. Processing control totals

d. Programmed exception reporting

175. An organizational control over CBIS operations is

a. Run-to-run balancing of control totals

b. Check digit verification of unique identifiers

c. Separation of operating and programming functions

d. Maintenance of output distribution logs

176. Which of the following methods of testing application controls

utilizes a generalized audit software package prepared by the

auditors?

a. Parallel simulation

b. Integrated testing facility approach

c. Test data approach

d. Exception report tests

177. An unauthorized employee took computer printouts from output

bins accessible to all employees. A control which would have

prevented this occurrence is

a. A storage/retention control

b. A spooler file control

c. An output review control

d. A report distribution control

178. Which of the following is a disadvantage of the integrated test

facility approach?

a. In establishing fictitious entities, the auditor may be

compromising audit independence.

b. Removing the fictitious transactions from the system

is somewhat difficult and, if not done carefully, may

contaminate the client's files.

c. ITF is simply an automated version of auditing

"around" the computer.

d. The auditor may not always have a current copy of

the authorized version of the client's program.

179. Totals of amounts in computer-record data fields which are not

usually added for other purposes but are used only for data

processing control purposes are called

a. Record totals

b. Hash totals

c. Processing data totals

d. Field totals

180. A hash total of employee numbers is part of the input to a

payroll master file update program. The program compares the

hash total to the total computed for transactions applied to

the master file. The purpose of this procedure is to:

a. Verify that employee numbers are valid

b. Verify that only authorized employees are paid

c. Detect errors in payroll calculations

d. Detect the omission of transaction processing

181. Matthews Corp. has changed from a system of recording time

worked on clock cards to a computerized payroll system in

which employees record time in and out with magnetic cards.

The CBIS automatically updates all payroll records. Because

of this change

a. A generalized computer audit program must be used

b. Part of the audit trail is altered

c. The potential for payroll related fraud is diminished

d. Transactions must be processed in batches

182. Generalized audit software is of primary interest to the auditor

in terms of its capability to

a. Access information stored on computer files

b. Select a sample of items for testing

c. Evaluate sample test results

d. Test the accuracy of the client's calculations

183. Accounts payable program posted a payable to a vendor not

included in the on-line vendor master file. A control which

would prevent this error is a

a. Validity check

b. Range check

c. Reasonableness test

d. Parity check

184. In a computerized sales processing system, which of the

following controls is most effective in preventing sales invoice

pricing errors?

a. Sales invoices are reviewed by the product

managers before being mailed to customers

b. Current sales prices are stored in the computer, and,

as stock numbers are entered from sales orders, the

computer automatically prices the orders

c. Sales prices, as well as product numbers, are

entered as sales orders are entered at remote

terminal locations

d. Sales prices are reviewed and updated on a

quarterly basis

185. Which of the following is likely to be of least importance to an

auditor in reviewing the internal control in a company with a

CBIS?

a. The segregation of duties within the data processing

center.

b. The control over source documents

c. The documentation maintained for accounting

applications.

d. The cost/benefit ratio of data processing operations

186. For the accounting system of Acme Company, the amounts of

cash disbursements entered into an CBIS terminal are

transmitted to the computer that immediately transmits the

amounts back to the terminal for display on the terminal screen.

This display enables the operator to

a. Establish the validity of the account number

b. Verify the amount was entered accurately

c. Verify the authorization of the disbursement

d. Prevent the overpayment of the account

187. Which of the following audit techniques most likely would

provide an auditor with the most assurance about the

effectiveness of the operation of an internal control procedure?

a. Inquiry of client personnel

b. Recomputation of account balance amounts

c. Observation of client personnel

d. Confirmation with outside parties

188. Adequate technical training and proficiency as an auditor

encompasses an ability to understand a CBIS sufficiently to

identify and evaluate

a. The processing and imparting of information

b. Essential accounting control features

c. All accounting control features

d. The degree to which programming conforms with

application of generally accepted accounting

principles.

189. Which of the following is not a major reason why an accounting

audit trail should be maintained for a computer system?

a. Query answering

b. Deterrent to fraud

c. Monitoring purposes

d. Analytical review

190. Adequate control over access to data processing is required to

a. Prevent improper use or manipulation of data files

and programs

b. Ensure that only console operators have access to

program documentation

c. Minimize the need for backup data files

d. Ensure that hardware controls are operating

effectively and as designed by the computer

manufacturer

191. When testing a computerized accounting system, which of the

following is not true of the test data approach?

a. The test data need consist of only those valid and

invalid conditions in which the auditor is interested

b. Only one transaction of each type need be tested

c. Test data are processed by the client's computer

programs under the auditor's control

d. The test data must consist of all possible valid and

invalid conditions

192. In studying a client's internal controls, an auditor must be able

to distinguish between prevention controls and detection

controls. Of the following data processing controls, which is the

best detection control?

a. Use of data encryption techniques

b. Review of machine utilization logs

c. Policy requiring password security

d. Backup and recovery procedure

193. Which of the following procedures is an example of auditing

"around" the computer?

a. The auditor traces adding machine tapes of sales

order batch totals to a computer printout of the sales

journal

b. The auditor develops a set of hypothetical sales

transactions and, using the client's computer

program, enters the transactions into the system and

observes the processing flow

c. The auditor enters hypothetical transactions into the

client's processing system during client processing of

live" data

d. The auditor observes client personnel as they

process the biweekly payroll. The auditor is primarily

concerned with computer rejection of data that fails

to meet reasonableness limits

194. Auditing by testing the input and output of a computer-based

system instead of the computer program itself will

a. Not detect program errors which do not show up in

the output sampled

b. Detect all program errors, regardless of the nature of

the output

c. Provide the auditor with the same type of evidence

d. Not provide the auditor with confidence in the results

of the auditing procedures

195. Which of the following is an acknowledged risk of using test

data when auditing CBIS records?

a. The test data may not include all possible types of

transactions

b. The computer may not process a simulated

transaction in the same way it would an identical

actual transaction

c. The method cannot be used with simulated master

records

d. Test data may be useful in verifying the correctness

of account balances, but not in determining the

presence of processing controls

196. When the auditor encounters sophisticated computer-based

systems, he or she may need to modify the audit approach. Of

the following conditions, which one is not a valid reason for

modifying the audit approach?

a. More advanced computer systems produce less

documentation, thus reducing the visibility of the

audit trail

b. In complex comuter-based systems, computer

verification of data at the point of input replaces the

manual verification found in less sophisticated data

processing systems

c. Integrated data processing has replaced the more

traditional separation of duties that existed in manual

and batch processing systems.

d. Real-time processing of transactions has enabled the

auditor to concentrate less on the completeness

assertion

197. If a control total were to be computed on each of the following

data items, which would best be identified as a hash total for a

payroll CBIS application?

a. Net pay

b. Department numbers

c. Hours worked

d. Total debits and total credits

198. In a distributed data base (DDB) environment, control tests for

access control administration can be designed which focus on

a. Reconciliation of batch control totals

b. Examination of logged activity

c. Prohibition of random access

d. Analysis of system generated core dumps

199. A control to verify that the dollar amounts for all debits and

credits for incoming transactions are posted to a receivables

master file is the:

a. Generation number check

b. Master reference check

c. Hash total

d. Control total

200. The program flowcharting symbol representing a decision is a

a. Triangle

b. Circle

c. Rectangle

d. Diamond

201. An update program for bank account balances calculates

check digits for account numbers. This is an example of

a. An input control

b. A file management control

c. Access control

d. An output control

202. CBIS controls are frequently classified as to general controls

and application controls. Which of the following is an example

of an application control?

a. Programmers may access the computer only for

testing and "debugging" programs

b. All program changes must be fully documented and

approved by the information systems manager and

the user department authorizing the change

c. A separate data control group is responsible for

distributing output, and also compares input and

output on a test basis

d. In processing sales orders, the computer compares

customer and product numbers with internally stored

lists

203. After a preliminary phase of the review of a client's CBIS

controls, an auditor may decide not to perform further tests

related to the control procedures within the CBIS portion of the

client's internal control system. Which of the following would

not be a valid reason for choosing to omit further testing?

a. The auditor wishes to further reduce assessed risk

b. The controls duplicate operative controls existing

elsewhere in the system

c. There appear to be major weaknesses that would

preclude reliance on the stated procedures

d. The time and dollar costs of testing exceed the time

and dollar savings in substantive testing if the

controls are tested for compliance

204. For good internal control over computer program changes, a

policy should be established requiring that

a. The programmer designing the change adequately

test the revised program

b. All program changes be supervised by the CBIS

control group

c. Superseded portions of programs be deleted from

the program run manual to avoid confusion

d. All proposed changes be approved in writing by a

responsible individual.

205. Which of the following is not a technique for testing data

processing controls?

a. The auditor develops a set of payroll test data that

contain numerous errors. The auditor plans to enter

these transactions into the client's system and

observe whether the computer detects and properly

responds to the error conditions

b. The auditor utilizes the computer to randomly select

customer accounts for confirmation

c. The auditor creates a set of fictitious custom

accounts and introduces hypothetical sales

transactions, as well as sales returns and

allowances, simultaneously with the client's live data

processing

d. At the auditor's request, the client has modified its

payroll processing program so as to separately

record any weekly payroll entry consisting of 60

hours or more. These separately recorded

("marked") entries are locked into the system and are

available only to the auditor

206. Which of the following would lessen internal control in a CBIS?

a. The computer librarian maintains custody of

computer program instructions and detailed listings

b. Computer operators have access to operator

instructions and detailed program listings

c. The control group is solely responsible for the

distribution of all computer output

d. Computer programmers write and debug programs

which perform routines designed by the systems

analyst

207. Access control in an on-line CBIS can best be provided in

most circumstances by

a. An adequate librarianship function controlling access

to files

b. A label affixed to the outside of a file medium holder

that identifies the contents

c. Batch processing of all input through a centralized,

well-guarded facility

d. User and terminal identification controls, such as

passwords

208. While entering data into a cash receipts transaction file, an

employee transposed two numbers in a customer code. Which

of the following controls could prevent input of this type of

error?

a. Sequence check

b. Record check

c. Self-checking digit

d. Field-size check

209. What is the computer process called when data processing is

performed concurrently with a particular activity and the results

are available soon enough to influence the particular course of

action being taken or the decision being made?

a. Batch processing

b. Real time processing

c. Integrated data processing

d. Random access processing

210. Reconciling processing control totals is an example of

a. An input control

b. An output control

c. A processing control

d. A file management control

211. Disadvantage of auditing around the computer is that it

a. Permits no assessment of actual processing

b. Requires highly skilled auditors

c. Demands intensive use of machine resources

d. Interacts actively with auditee applications

212. The completeness of computer-generated sales figures can be

tested by comparing the number of items listed on the daily

sales report with the number of items billed on the actual

invoices. This process uses

a. Check digits

b. Control totals

c. Validity tests

d. Process tracing data

213. Which of the following controls would be most efficient in

reducing common data input errors?

a. Keystroke verification

b. A set of well-designed edit checks

c. Balancing and reconciliation

d. Batch totals

214. On-line real-time systems and electronic data interchange

systems have the advantages of providing more timely

information and reducing the quantity of documents associated

with less automated systems. The advantages, however, may

create some problems for the auditor. Which of the following

characteristics of these systems does not create an audit

problem?

a. The lack of traditional documentation of transactions

creates a need for greater attention to programmed

controls at the point of transaction input

b. Hard copy may not be retained by the client for long

periods of time, thereby necessitating more frequent

visits by the auditor

c. Control testing may be more difficult given the

increased vulnerability of the client's files to

destruction during the testing process

d. Consistent on-line processing of recurring data

increases the incidence of errors

215. Creating simulated transactions that are processed through a

system to generate results that are compared with

predetermined results, is an auditing procedure referred to as

a. Desk checking

b. Use of test data

c. Completing outstanding jobs

d. Parallel simulation

216. To obtain evidential matter about control risk, an auditor

ordinarily selects tests from a variety of techniques, including

a. Analysis

b. Confirmations

c. Reprocessing

d. Comparison

217. A major exposure associated with the rapidly expanding use of

microcomputers is the absence of:

a. Adequate size of main memory and disk storage

b. Compatible operating systems

c. Formalized procedures for purchase justification

d. Physical, data file, and program security

218. To ensure that goods received are the same as those shown

on the purchase invoice, a computerized system should:

a. Match selected fields of the purchase invoice to

goods received

b. Maintain control totals of inventory value

c. Calculate batch totals for each input

d. Use check digits in account numbers

219. Errors in data processed in a batch computer system may not

be detected immediately because

a. Transaction trails in a batch system are available

only for a limited period of time

b. There are time delays in processing transactions in a

batch system

c. Errors in some transactions cause rejection of other

transactions in the batch

d. Random errors are more likely in a batch system

than in an on-line system

220. Which of the following is a computer test made to ascertain

whether a given characteristic belongs to the group?

a. Parity check

b. Validity check

c. Echo check

d. Limit check.

Unformatted Attachment Preview

AUDITING IN A COMPUTERIZED ENVIRONMENT 1. Which statement is incorrect when auditing in a CIS environment? a. A CIS environment exists when a computer of any type or size is involved in the processing by the entity of financial information of significance to the audit, whether that computer is operated by the entity or by a third party. b. The auditor should consider how a CIS environment affects the audit. c. The use of a computer changes the processing, storage and communication of financial information and may affect the accounting and internal control systems employed by the entity. d. A CIS environment changes the overall objective and scope of an audit. 2. Which of the following standards or group of standards is mostly affected by a computerized information system environment? a. General standards b. Reporting standards c. Second standard of field work d. Standards of fieldwork 3. Which of the following is least considered if the auditor has to determine whether specialized CIS skills are needed in an audit? a. The auditor needs to obtain a sufficient understanding of the accounting and internal control system affected by the CIS environment. b. The auditor needs to determine the effect of the CIS environment on the assessment of overall risk and of risk at the account balance and class of transactions level. c. Design and perform appropriate tests of controls and substantive procedures. d. The need of the auditor to make analytical procedures during the completion stage of audit. 4. 5. 6. 7. It relates to materiality of the financial statement assertions affected by the computer processing. a. Threshold b. Relevance c. Complexity d. Significance Which of the following least likely indicates a complexity of computer processing? a. Transactions are exchanged electronically with other organizations without manual review of their propriety. b. The volume of the transactions is such that users would find it difficult to identify and correct errors in processing. c. The computer automatically generates material transactions or entries directly to another applications. d. The system generates a daily exception report The nature of the risks and the internal characteristics in CIS environment that the auditors are mostly concerned include the following except: a. Lack of segregation of functions. b. Lack of transaction trails. c. Dependence of other control over computer processing. d. Cost-benefit ratio. Which of the following is least likely a risk characteristic associated with CIS environment? a. Errors embedded in an application’s program logic maybe difficult to manually detect on a timely basis. b. Many control procedures that would ordinarily be performed by separate individuals in manual system maybe concentrated in CIS. c. d. The potential unauthorized access to data or to alter them without visible evidence maybe greater. Initiation of changes in the master file is exclusively handled by respective users. 8. Which of the following significance and complexity of the CIS activities should an auditor least understand? a. The organizational structure of the client’s CIS activities. b. Lack of transaction trails. c. The significance and complexity of computer processing in each significant accounting application. d. The use of software packages instead of customized software. 9. Which statement is correct regarding personal computer systems? a. Personal computers or PCs are economical yet powerful self-contained general purpose computers consisting typically of a central processing unit (CPU), memory, monitor, disk drives, printer cables and modems. b. Programs and data are stored only on nonremovable storage media. c. Personal computers cannot be used to process accounting transactions and produce reports that are essential to the preparation of financial statements. d. Generally, CIS environments in which personal computers are used are the same with other CIS environments. 10. A personal computer can be used in various configurations, including a. A stand-alone workstation operated by a single user or a number of users at different times. b. A workstation which is part of a local area network of personal computers. c. A workstation connected to a server. d. All of the above. 11. Which statement is incorrect regarding personal computer configurations? a. The stand-alone workstation can be operated by a single user or a number of users at different times accessing the same or different programs. b. A stand-alone workstation may be referred to as a distributed system. c. A local area network is an arrangement where two or more personal computers are linked together through the use of special software and communication lines. d. Personal computers can be linked to servers and used as part of such systems, for example, as an intelligent on-line workstation or as part of a distributed accounting system. 12. Which of the following is the least likely characteristic of personal computers? a. They are small enough to be transportable. b. They are relatively expensive. c. They can be placed in operation quickly. d. The operating system software is less comprehensive than that found in larger computer environments. 13. Which of the following is an inherent characteristic of software package? a. They are typically used without modifications of the programs. b. The programs are tailored-made according to the specific needs of the user. c. They are developed by software manufacturer according to a particular user’s specifications. d. It takes a longer time of implementation. 14. Which of the following is not normally a removable storage media? a. Compact disk b. Tapes c. Diskettes d. Hard disk 15. It is a computer program (a block of executable code) that attaches itself to a legitimate program or data file and uses itself as a transport mechanism to reproduce itself without the knowledge of the user. a. Virus b. System management program c. Utility program d. Encryption 16. Which statement is incorrect regarding internal control in personal computer environment? a. Generally, the CIS environment in which personal computers are used is less structured than a centrally-controlled CIS environment. b. Controls over the system development process and operations may not be viewed by the developer, the user or management as being as important or costeffective. c. In almost all commercially available operating systems, the built-in security provided has gradually increased over the years. d. In a typical personal computer environment, the distinction between general CIS controls and CIS application controls is easily ascertained. 17. Personal computers are susceptible to theft, physical damage, unauthorized access or misuse of equipment. Which of the following is least likely a physical security to restrict access to personal computers when not in use? a. Using door locks or other security protection during non-business hours. b. Fastening the personal computer to a table using security cables. c. Locking the personal computer in a protective cabinet or shell. d. Using anti-virus software programs. 18. Which of the following is not likely a control over removable storage media to prevent misplacement, alteration without authorization or destruction? a. Using cryptography, which is the process of transforming programs and information into an unintelligible form. b. Placing responsibility for such media under personnel whose responsibilities include duties of software custodians or librarians. c. Using a program and data file check-in and checkout system and locking the designated storage locations. d. Keeping current copies of diskettes, compact disks or back-up tapes and hard disks in a fireproof container, either on-site, off-site or both. 19. 19. Which of the following least likely protects critical and sensitive information from unauthorized access in a personal computer environment? a. Using secret file names and hiding the files. b. Keeping of back up copies offsite. c. Employing passwords. d. Segregating data into files organized under separate file directories. 20. It refers to plans made by the entity to obtain access to comparable hardware, software and data in the event of their failure, loss or destruction. a. Back-up b. Encryption c. Anti-virus d. Wide Area Network (WAN) 21. The effect of personal computers on the accounting system and the associated risks will least likely depend on a. The extent to which the personal computer is being used to process accounting applications. b. The type and significance of financial transactions being processed. c. The nature of files and programs utilized in the applications. d. The cost of personal computers. 22. The auditor may often assume that control risk is high in personal computer systems since, it may not be practicable or cost-effective for management to implement sufficient controls to reduce the risks of undetected errors to a minimum level. This least likely entail a. More physical examination and confirmation of assets. b. More analytical procedures than tests of details. c. Larger sample sizes. d. Greater use of computer-assisted audit techniques, where appropriate. 23. Computer systems that enable users to access data and programs directly through workstations are referred to as a. On-line computer systems b. Personal computer systems c. Database management systems (DBMS) d. Database systems 24. On-line systems allow users to initiate various functions directly. Such functions include: I. Entering transactions II. Requesting reports III. Making inquiries IV. Updating master files a. I, II, III and IV b. I and II c. I, II and III d. I and IV 25. Many different types of workstations may be used in on-line computer systems. The functions performed by these workstations least likely depend on their a. Logic b. Transmission c. Storage d. Cost 26. Types of workstations include General Purpose Terminals and Special Purpose Terminals. Special Purpose Terminals include a. Basic keyboard and monitor b. Point of sale devices c. Intelligent terminal d. Personal computers 27. Special Purpose Terminal used to initiate, validate, record, transmit and complete various banking transactions a. Automated teller machines b. Intelligent terminal c. Point of sale devices d. Personal computers 28. Which statement is incorrect regarding workstations? a. Workstations may be located either locally or at remote sites. b. Local workstations are connected directly to the computer through cables. c. Remote workstations require the use of telecommunications to link them to the computer. d. Workstations cannot be used by many users, for different purposes, in different locations all at the same time. 29. On-line computer systems may be classified according to a. How information is entered into the system. b. How it is processed. c. When the results are available to the user. d. All of the above. 30. In an on-line/real time processing system a. Individual transactions are entered at workstations, validated and used to update related computer files immediately. b. Individual transactions are entered at a workstation, subjected to certain validation checks and added to a transaction file that contains other transactions entered during the period. c. Individual transactions immediately update a memo file containing information which has been extracted from the most recent version of the master file. d. The master files are updated by other systems. 31. It combines on-line/real time processing and on-line/batch processing. a. On-Line/Memo Update (and Subsequent Processing) b. On-Line Downloading/Uploading Processing c. On-Line/Inquiry d. On-Line/Combined Processing 32. It is a communication system that enables computer users to share computer equipment, application software, data and voice and video transmissions. a. Network b. File server c. Host d. Client 33. A type of network that multiple buildings are close enough to create a campus, but the space between the buildings is not under the control of the company is a. Local Area Network (LAN) b. Metropolitan Area Network (MAN) c. Wide Area Network (WAN) d. World Wide Web (WWW) 34. Which of the following is least likely a characteristic of Wide Area Network (WAN)? a. Created to connect two or more geographically separated LANs. b. Typically involves one or more long-distance providers, such as a telephone company to provide the connections. c. WAN connections tend to be faster than LAN. d. Usually more expensive than LAN. 35. Gateway is a. A hardware and software solution that enables communications between two dissimilar networking systems or protocols. b. A device that forwards frames based on destination addresses. c. A device that connects and passes packets between two network segments that use the same communication protocol. d. A device that regenerates and retransmits the signal on a network. 36. A device that works to control the flow of data between two or more network segments a. Bridge b. Router c. Repeater d. Switch 37. The undesirable characteristics of on-line computer systems least likely include a. Data are usually subjected to immediate validation checks. b. Unlimited access of users to all of the functions in a particular application. c. Possible lack of visible transaction trail. d. Potential programmer access to the system. 38. Certain general CIS controls that are particularly important to on-line processing least likely include a. Access controls. b. System development and maintenance controls. c. Edit, reasonableness and other validation tests. d. Use of anti-virus software program. 39. Certain CIS application controls that are particularly important to on-line processing least likely include a. Pre-processing authorization. b. Transaction logs. c. Cut-off procedures. d. Balancing. 40. Risk of fraud or error in on-line systems may be reduced in the following circumstances, except a. If on-line data entry is performed at or near the point where transactions originate, there is less risk that the transactions will not be recorded. b. If invalid transactions are corrected and re-entered immediately, there is less risk that such transactions will not be corrected and re-submitted on a timely basis. c. If data entry is performed on-line by individuals who understand the nature of the transactions involved, the data entry process may be less prone to errors than when it is performed by individuals unfamiliar with the nature of the transactions. d. On-line access to data and programs through telecommunications may provide greater opportunity for access to data and programs by unauthorized persons. 41. Risk of fraud or error in on-line computer systems may be increased for the following reasons, except a. If workstations are located throughout the entity, the opportunity for unauthorized use of a workstation and the entry of unauthorized transactions may increase. b. Workstations may provide the opportunity for unauthorized uses such as modification of previously entered transactions or balances. c. If on-line processing is interrupted for any reason, for example, due to faulty telecommunications, there may be a greater chance that transactions or files may be lost and that the recovery may not be accurate and complete. d. If transactions are processed immediately on-line, there is less risk that they will be processed in the wrong accounting period. 42. 42. The following matters are of particular importance to the auditor in an on-line computer system, except a. Authorization, completeness and accuracy of on-line transactions. b. Integrity of records and processing, due to on-line access to the system by many users and programmers. c. Changes in the performance of audit procedures including the use of CAAT's. d. Cost-benefit ratio of installing on-line computer system. 43. A collection of data that is shared and used by a number of different users for different purposes. a. Database b. Information file c. Master file d. Transaction file 44. Which of the following is least likely a characteristic of a database system? a. Individual applications share the data in the database for different purposes. b. Separate data files are maintained for each application and similar data used by several applications may be repeated on several different files. c. A software facility is required to keep track of the location of the data in the database. d. Coordination is usually performed by a group of individuals whose responsibility is typically referred to as "database administration." 45. Database administration tasks typically include I. Defining the database structure. II. Maintaining data integrity, security and completeness. III. Coordinating computer operations related to the database. IV. Monitoring system performance. V. Providing administrative support. a. All of the above b. All except I c. II and V only d. II, III and V only 46. Due to data sharing, data independence and other characteristics of database systems a. General CIS controls normally have a greater influence than CIS application controls on database systems. b. CIS application controls normally have a greater influence than general CIS controls on database systems. c. General CIS controls normally have an equal influence with CIS application controls on database systems. d. CIS application controls normally have no influence on database systems. 50. The effect of a database system on the accounting system and the associated risks will least likely depend on: a. The extent to which databases are being used by accounting applications. b. The type and significance of financial transactions being processed. c. The nature of the database, the DBMS, the database administration tasks and the applications. d. The CIS application controls. 51. Audit procedures in a database environment will be affected principally by a. The extent to which the data in the database are used by the accounting system. b. The type and significance of financial transactions being processed. c. The nature of the database, the DBMS, the database administration tasks and the applications. d. The general CIS controls which are particularly important in a database environment. 52. Which statement is incorrect regarding the characteristics of a CIS organizational structure? a. Certain data processing personnel may be the only ones with a detailed knowledge of the interrelationship between the source of data, how it is processed and the distribution and use of the output. b. Many conventional controls based on adequate segregation of incompatible functions may not exist, or in the absence of access and other controls, may be less effective. c. Transaction and master file data are often concentrated, usually in machine-readable form, either in one computer installation located centrally or in a number of installations distributed throughout an entity. d. Systems employing CIS methods do not include manual operations since the number of persons involved in the processing of financial information is significantly reduced. 47. Which statement is incorrect regarding the general CIS controls of particular importance in a database environment? a. Since data are shared by many users, control may be enhanced when a standard approach is used for developing each new application program and for application program modification. b. Several data owners should be assigned responsibility for defining access and security rules, such as who can use the data (access) and what functions they can perform (security). c. User access to the database can be restricted through the use of passwords. d. Responsibilities for performing the various activities required to design, implement and operate a database are divided among technical, design, administrative and user personnel. 53. System characteristics that may result from the nature of CIS processing include, except a. Absence of input documents. b. Lack of visible transaction trail. c. Lack of visible output. d. Difficulty of access to data and computer programs. 48. These require a database administrator to assign security attributes to data that cannot be changed by database users. a. Discretionary access controls b. Name-dependent restrictions c. Mandatory access controls d. Content-dependent restrictions. 55. Which statement is incorrect regarding internal controls in a CIS environment? a. Manual and computer control procedures comprise the overall controls affecting the CIS environment (general CIS controls) and the specific controls over the accounting applications (CIS application controls). b. The purpose of general CIS controls is to establish a framework of overall control over the CIS activities and to provide a reasonable level of assurance that the overall objectives of internal control are achieved. c. The purpose of CIS application controls is to establish specific control procedures over the application systems in order to provide reasonable assurance that all transactions are authorized and recorded, and are processed completely, accurately and on a timely basis. 49. A discretionary access control wherein users are permitted or denied access to data resource depending on the time series of accesses to and actions they have undertaken on data resources. a. Name-dependent restrictions b. Context-dependent restriction c. Content-dependent restriction d. History-dependent restriction 54. The development of CIS will generally result in design and procedural characteristics that are different from those found in manual systems. These different design and procedural aspectsof CIS include, except: a. Consistency of performance. b. Programmed control procedures. c. Vulnerability of data and program storage media d. Multiple transaction update of multiple computer files or databases. d. The internal controls over computer processing, which help to achieve the overall objectives of internal control, include only the procedures designed into computer programs. 56. General CIS controls may include, except: a. Organization and management controls. b. Delivery and support controls. c. Development and maintenance controls. d. Controls over computer data files. 57. 57. CIS application controls include, except a. Controls over input. b. Controls over processing and computer data files. c. Controls over output. d. Monitoring controls. 58. Which statement is incorrect regarding the review of general CIS controls and CIS application controls? a. The auditor should consider how these general CIS controls affect the CIS applications significant to the audit. b. General CIS controls that relate to some or all applications are typically interdependent controls in that their operation is often essential to the effectiveness of CIS application controls. c. Control over input, processing, data files and output may be carried out by CIS personnel, by users of the system, by a separate control group, or may be programmed into application software. d. It may be more efficient to review the design of the application controls before reviewing the general controls. 59. Which statement is incorrect regarding the evaluation of general CIS controls and CIS application controls? a. The general CIS controls may have a pervasive effect on the processing of transactions in application systems. b. If general CIS controls are not effective, there may be a risk that misstatements might occur and go undetected in the application systems. c. Manual procedures exercised by users may provide effective control at the application level. d. Weaknesses in general CIS controls cannot preclude testing certain CIS application controls. 60. The applications of auditing procedures using the computer as an audit tool refer to a. Integrated test facility b. Auditing through the computer c. Data-based management system d. Computer assisted audit techniques 61. Which statement is incorrect regarding CAATs? a. CAATs are often an efficient means of testing a large number of transactions or controls over large populations. b. To ensure appropriate control procedures, the presence of the auditor is not necessarily required at the computer facility during the running of a CAAT. c. The general principles outlined in PAPS 1009 apply in small entity IT environments. d. Where smaller volumes of data are processed, the use of CAATs is more cost effective. 62. Consists of generalized computer programs designed to perform common audit tasks or standardized data processing functions. a. Package or generalized audit software b. Utility programs c. Customized or purpose-written programs d. System management programs 63. Audit automation least likely include a. Expert systems. b. Tools to evaluate a client’s risk management procedures. c. Manual working papers. d. Corporate and financial modeling programs for use as predictive audit tests. 64. An internal auditor noted the following points when conducting a preliminary survey in connection with the audit of an EDP department. Which of the following would be considered a safeguard in the control system on which the auditor might rely? a. Programmers and computer operators correct daily processing problems as they arise. b. The control group works with user organizations to correct rejected input. c. New systems are documented as soon as possible after they begin processing live data. d. The average tenure of employees working in the EDP department is ten months. 65. An on-line access control that checks whether the user’s code number is authorized to initiate a specific type of transaction or inquiry is referred to as a. Password b. Compatibility test c. Limit check d. Reasonableness test 66. A control procedure that could be used in an on-line system to provide an immediate check on whether an account number has been entered on a terminal accurately is a a. Compatibility test b. Record count c. Hash total d. Self-checking digit 67. A control designed to catch errors at the point of data entry is a. Batch total b. Self-checking digit c. Record count d. Checkpoints 68. Program documentation is a control designed primarily to ensure that a. Programmers have access to the tape library or information on disk files. b. Programs do not make mathematical errors. c. Programs are kept up to date and perform as intended. d. Data have been entered and processed. 69. Some of the more important controls that relate to automated accounting information systems are validity checks, limit checks, field checks, and sign tests. These are classified as a. Control total validation routines b. Output controls c. Hash totaling d. Input validation routines 70. Most of today’s computer systems have hardware controls that are built in by the computer manufacturer. Common hardware controls are a. Duplicate circuitry, echo check, and internal header labels b. Tape file protection, cryptographic protection, and limit checks c. Duplicate circuitry, echo check, and dual reading d. Duplicate circuitry, echo check, tape file protection, and internal header labels 71. Computer manufacturers are now installing software programs permanently inside the computer as part of its main memory to provide protection from erasure or loss if there is interrupted electrical power. This concept is known as a. File integrity b. Random access memory (RAM) c. Software control d. Firmware 72. Which one of the following represents a lack of internal control in a computer-based information system? a. The design and implementation is performed in accordance with management’s specific authorization. b. Any and all changes in application programs have the authorization and approval of management. c. Provisions exist to protect data files from unauthorized access, modification, or destruction. d. Both computer operators and programmers have unlimited access to the programs and data files. 73. In an automated payroll processing environment, a department manager substituted the time card for a terminated employee with a time card for a fictitious employee. The fictitious employee had the same pay rate and hours worked as the terminated employee. The best control technique to detect this action using employee identification numbers would be a a. Batch total b. Hash total c. Record count d. Subsequent check 74. An employee in the receiving department keyed in a shipment from a remote terminal and inadvertently omitted the purchase order number. The best systems control to detect this error would be a. Batch total b. Sequence check c. Completeness test d. Reasonableness test 75. The reporting of accounting information plays a central role in the regulation of business operations. Preventive controls are an integral part of virtually all accounting processing systems, and much of the information generated by the accounting system is used for preventive control purposes. Which one of the following is not an essential element of a sound preventive control system? a. Separation of responsibilities for the recording, custodial, and authorization functions. b. Sound personnel policies. c. Documentation of policies and procedures. d. Implementation of state-of-the-art software and hardware. 76. The most critical aspect regarding separation of duties within information systems is between a. Project leaders and programmers b. Programmers and systems analysts c. Programmers and computer operators d. Data control and file librarians 77. Whether or not a real time program contains adequate controls is most effectively determined by the use of a. Audit software b. A tracing routine c. An integrated test facility d. A traditional test deck 78. Compatibility tests are sometimes employed to determine whether an acceptable user is allowed to proceed. In order to perform compatibility tests, the system must maintain an access control matrix. The one item that is not part of an access control matrix is a a. List of all authorized user code numbers and passwords. b. List of all files maintained on the system. c. d. Record of the type of access to which each user is entitled. Limit on the number of transaction inquiries that can be made by each user in a specified time period. 79. Which one of the following input validation routines is not likely to be appropriate in a real time operation? a. Field check b. Sequence check c. Sign check d. Redundant data check 80. Which of the following controls is a processing control designed to ensure the reliability and accuracy of data processing? a. b. c. d. Limit test Yes No No Yes Validity check test Yes No Yes No 81. Which of the following characteristics distinguishes computer processing from manual processing? a. Computer processing virtually eliminates the occurrence of computational error normally associated with manual processing. b. Errors or irregularities in computer processing will be detected soon after their occurrences. c. The potential for systematic error is ordinarily greater in manual processing than in computerized processing. d. Most computer systems are designed so that transaction trails useful for audit do not exist. 82. Which of the following most likely represents a significant deficiency in the internal control structure? a. The systems analyst review applications of data processing and maintains systems documentation. b. The systems programmer designs systems for computerized applications and maintains output controls. c. The control clerk establishes control over data received by the EDP department and reconciles control totals after processing d. The accounts payable clerk prepares data for computer processing and enters the data into the computer. 83. Which of the following activities would most likely be performed in the EDP Department? a. Initiation of changes to master records. b. Conversion of information to machine-readable form. c. Correction of transactional errors. d. Initiation of changes to existing applications. 84. For control purposes, which of the following should be organizationally segregated from the computer operations function? a. Data conversion b. Systems development c. Surveillance of CRT messages d. Minor maintenance according to a schedule 85. Which of the following is not a major reason for maintaining an audit trail for a computer system? a. Deterrent to irregularities b. Analytical procedures c. Monitoring purposes d. Query answering 86. In an automated payroll system, all employees in the finishing department were paid the rate of P75 per hour when the authorized rate was P70 per hour. Which of the following controls would have been most effective in preventing such an error? a. Access controls which would restrict the personnel department’s access to the payroll master file data. b. c. d. A review of all authorized pay rate changes by the personnel department. The use of batch control totals by department. A limit test that compares the pay rates per department with the maximum rate for all employees. 87. Which of the following errors would be detected by batch controls? a. A fictitious employee as added to the processing of the weekly time cards by the computer operator. b. An employee who worked only 5 hours in the week was paid for 50 hours. c. The time card for one employee was not processed because it was lost in transit between the payroll department and the data entry function. d. All of the above. 88. The use of a header label in conjunction with magnetic tape is most likely to prevent errors by the a. Computer operator b. Computer programmer c. Keypunch operator d. Maintenance technician 89. For the accounting system of ACME Company, the amounts of cash disbursements entered into an EDP terminal are transmitted to the computer that immediately transmits the amounts back to the terminal for display on the terminal screen. This display enables the operator to a. Establish the validity of the account number b. Verify the amount was entered accurately c. Verify the authorization of the disbursements d. Prevent the overpayment of the account 90. When EDP programs or files can be accessed from terminals, users should be required to enter a(an) a. Parity check b. Self-diagnostic test c. Personal identification code d. Echo check 91. The possibility of erasing a large amount of information stored on magnetic tape most likely would be reduced by the use of a. File protection ring b. Completeness tests c. Check digits d. Conversion verification 92. Which of the following controls most likely would assure that an entity can reconstruct its financial records? a. Hardware controls are built into the computer by the computer manufacturer. b. Backup diskettes or tapes of files are stored away from originals. c. Personnel who are independent of data input perform parallel simulations. d. System flowcharts provide accurate descriptions of input and output operations. 93. Mill Co. uses a batch processing method to process its sales transactions. Data on Mill’s sales transaction tape are electronically sorted by customer number and are subject to programmed edit checks in preparing its invoices, sales journals, and updated customer account balances. One of the direct outputs of the creation of this tape most likely would be a a. Report showing exceptions and control totals. b. Printout of the updated inventory records. c. Report showing overdue accounts receivable. d. Printout of the sales price master file. 94. Using microcomputers in auditing may affect the methods used to review the work of staff assistants because a. The audit field work standards for supervision may differ. b. Documenting the supervisory review may require assistance of consulting services personnel. c. d. Supervisory personnel may not have an understanding of the capabilities and limitations of microcomputers. Working paper documentation may not contain readily observable details of calculations. 95. An auditor anticipates assessing control risk at a low level in a computerized environment. Under these circumstances, on which of the following procedures would the auditor initially focus? a. Programmed control procedures b. Output control procedures c. Application control procedures d. General control procedures 96. After the preliminary phase of the review of a client’s EDP controls, an auditor may decide not to perform tests of controls (compliance tests) related to the control procedures within the EDP portion of the client’s internal control structure. Which of the following would not be a valid reason for choosing to omit such tests? a. The controls duplicate operative controls existing elsewhere in the structure. b. There appear to be major weaknesses that would preclude reliance on the stated procedure. c. The time and costs of testing exceed the time and costs in substantive testing if the tests of controls show the controls to be operative. d. The controls appear adequate. 97. Which of the following client electronic data processing (EDP) systems generally can be audited without examining or directly testing the EDP computer programs of the system? a. A system that performs relatively uncomplicated processes and produces detailed output. b. A system that affects a number of essential master files and produces a limited output. c. A system that updates a few essential master files and produces no printed output other than final balances. d. A system that performs relatively complicated processing and produces very little detailed output. 98. Computer systems are typically supported by a variety of utility software packages that are important to an auditor because they a. May enable unauthorized changes to data files if not properly controlled. b. Are very versatile programs that can be used on hardware of many manufacturers. c. May be significant components of a client’s application programs. d. Are written specifically to enable auditors to extract and sort data. 99. To obtain evidence that online access controls are properly functioning, an auditor most likely would a. Create checkpoints at periodic intervals after live data processing to test for unauthorized use of the system. b. Examine the transaction log to discover whether any transactions were lost or entered twice due to a system malfunction c. Enter invalid identification numbers or passwords to ascertain whether the system rejects them. d. Vouch a random sample of processed transactions to assure proper authorization 100. Which of the following statements most likely represents a disadvantage for an entity that keeps microcomputer-prepared data files rather than manually prepared files? a. Attention is focused on the accuracy of the programming process rather than errors in individual transactions. b. It is usually easier for unauthorized persons to access and alter the files. c. Random error associated with processing similar transactions in different ways is usually greater. d. It is usually more difficult to compare recorded accountability with physical count of assets. 101. An auditor would least likely use computer software to a. Access client data files b. Assess EDP controls c. Prepare spreadsheets d. Construct parallel simulations 102. A primary advantage of using generalized audit software packages to audit the financial statements of a client that uses an EDP system is that the auditor may a. Consider increasing the use of substantive tests of transactions in place of analytical procedures. b. Substantiate the accuracy of data through selfchecking digits and hash totals. c. Reduce the level of required tests of controls to a relatively small amount. d. Access information stored on computer files while having a limited understanding of the client’s hardware and software features. 103. Auditors often make use of computer programs that perform routine processing functions such as sorting and merging. These programs are made available by electronic data processing companies and others and are specifically referred to as a. Compiler programs b. Utility programs c. Supervisory programs d. User programs 104. Smith Corporation has numerous customers. A customer file is kept on disk storage. Each customer file contains name, address, credit limit, and account balance. The auditor wishes to test this file to determine whether the credit limits are being exceeded. The best procedure for the auditor to follow would be to a. Develop test data that would cause some account balances to exceed the credit limit and determine if the system properly detects such situations. b. Develop a program to compare credit limits with account balances and print out the details of any account with a balance exceeding its credit limit. c. Request a printout of all account balances so they can be manually checked against the credit limits. d. Request a printout of a sample of account balances so they can be individually checked against the credit limits. 105. The use of generalized audit software package a. Relieves an auditor of the typical tasks of investigating exceptions, verifying sources of information, and evaluating reports. b. Is a major aid in retrieving information from computerized files. c. Overcomes the need for an auditor to learn much about computers. d. Is a form of auditing around the computer. 106. An auditor used test data to verify the existence of controls in a certain computer program. Even though the program performed well on the test, the auditor may still have a concern that a. The program tested is the same one used in the regular production runs. b. Generalized audit software may have been a better tool to use. c. Data entry procedures may change and render the test useless. d. The test data will not be relevant in subsequent audit periods. 107. An auditor most likely would introduce test data into a computerized payroll system to test internal controls related to the a. Existence of unclaimed payroll checks held by supervisors. b. c. d. Early cashing of payroll checks by employees. Discovery of invalid employee I.D. numbers. Proper approval of overtime by supervisors. 108. When an auditor tests a computerized accounting system, which of the following is true of the test data approach? a. Test data must consist of all possible valid and invalid conditions. b. The program tested is different from the program used throughout the year by the client. c. Several transactions of each type must be tested. d. Test data are processed by the client’s computer programs under the auditor’s control. 109. Which of the following statements is not true to the test data approach when testing a computerized accounting system? a. The test need consist of only those valid and invalid conditions which interest the auditor b. Only one transaction of each type need be tested. c. The test data must consist of all possible valid and invalid conditions. d. Test data are processed by the client’s computer programs under the auditor’s control. 110. Which of the following is not among the errors that an auditor might include in the test data when auditing a client’s EDP system? a. Numeric characters in alphanumeric fields. b. Authorized code. c. Differences in description of units of measure. d. Illogical entries in fields whose logic is tested by programmed consistency checks. 111. An auditor who is testing EDP controls in a payroll system would most likely use test data that contain conditions such as a. Deductions not authorized by employees. b. Overtime not approved by supervisors. c. Time tickets with invalid job numbers. d. Payroll checks with unauthorized signatures. 112. Auditing by testing the input and output of an EDP system instead of the computer program itself will a. Not detect program errors which do not show up in the output sampled. b. Detect all program errors, regardless of the nature of the output. c. Provide the auditor with the same type of evidence. d. Not provide the auditor with confidence in the results of the auditing procedures. 113. Which of the following computer-assisted auditing techniques allows fictitious and real transactions to be processed together without client operating personnel being aware of the testing process? a. Integrated test facility b. Parallel simulation c. Input controls matrix d. Data entry monitor 114. Which of the following methods of testing application controls utilizes a generalized audit software package prepared by the auditors? a. Parallel simulation b. Test data approach c. Integrated testing facility approach d. Exception report tests 115. Misstatements in a batch computer system caused by incorrect programs or data may not be detected immediately because a. Errors in some transactions may cause rejection of other transactions in the batch. b. The identification of errors in input data typically is not part of the program. c. There are time delays in processing transactions in a batch system. d. The processing of transactions in a batch system is not uniform. 116. Which of the following is not a characteristic of a batch processed computer system? a. The collection of like transactions which are sorted and processed sequentially against a master file. b. Keypunching of transactions, followed by machine processing. c. The production of numerous printouts. d. The posting of a transaction, as it occurs, to several files, without immediate printouts. 117. Where disk files are used, the grandfather-father-son updating backup concept is relatively 118. difficult to implement because the a. Location of information points on disks is an extremely time consuming task. b. Magnetic fields and other environmental factors cause off-site storage to be impractical. c. Information must be dumped in the form of hard copy if it is to be reviewed before used in d. Process of updating old records is destructive. 119. An auditor would most likely be concerned with which of the following controls in a distributed data processing system? a. Hardware controls b. Access controls c. Systems documentation controls d. Disaster recovery controls 120. If a control total were computed on each of the following data items, which would best be identified as a hash total for a payroll EDP application? a. Total debits and total credits b. Department numbers c. Net pay d. Hours worked 121. Which of the following is a computer test made to ascertain whether a given characteristic belongs to the group? a. Parity check b. Echo check c. Validity check d. Limit check 122. A control feature in an electronic data processing system requires the central processing unit (CPU) to send signals to the printer to activate the print mechanism for each character. The print mechanism, just prior to printing, sends a signal back to the CPU verifying that the proper print position has been activated. This type of hardware control is referred to as a. Echo check b. Signal control c. Validity control d. Check digit control 123. Which of the following is an example of a check digit? a. An agreement of the total number of employees to the total number of checks printed by the computer. b. An algebraically determined number produced by the other digits of the employee number c. A logic test that ensures all employee numbers are nine digits. d. A limit check that an employee’s hours do not exceed 50 hours per work week. 124. In a computerized system, procedure or problem-oriented language is converted to machine language through a(an) a. Interpreter b. Verifier c. Compiler d. Converter 125. A customer erroneously ordered Item No. 86321 rather than item No. 83621. When this order is processed, the vendor’s EDP department would identify the error with what type of control? a. Key verifying b. Batch total c. Self-checking digit d. Item inspection 126. The computer process whereby data processing is performed concurrently with a particular activity and the results are available soon enough to influence the course of action being taken or the decision being made is called: a. Random access sampling b. On-line, real-time system c. Integrated data processing d. Batch processing system 127. Internal control is ineffective when computer department personnel a. Participate in computer software acquisition decisions. b. Design documentation for computerized systems. c. Originate changes in master file. d. Provide physical security for program files. 128. Test data, integrated test data and parallel simulation each require an auditor to prepare data and computer programs. CPAs who lack either the technical expertise or time to prepare programs should request from the manufacturers or EDP consultants for a. The program Code b. Generalized audit software c. Flowchart checks d. Application controls 129. Which of the following best describes a fundamental control weakness often associated with electronic data processing system? a. EDP equipment is more subject to system error than manual processing is subject to human error. b. Monitoring is not an adequate substitute for the use of test data. c. EDP equipment processes and records similar transactions in a similar manner. d. Functions that would normally be separated in a manual system are combined in the EDP system like the function of programmers and operators. 130. Which of the following tasks could not be performed when using a generalized audit software package? a. Selecting inventory items for observations. b. Physical count of inventories. c. Comparison of inventory test counts with perpetual records. d. Summarizing inventory turnover statistics for obsolescence analysis. 131. All of the following are “auditing through the computer” techniques except a. Reviewing source code b. Automated tracking and mapping c. Test-decking d. Integrated test facility 132. The output of a parallel simulation should always be a. Printed on a report. b. Compared with actual results manually. c. Compared with actual results using a comparison program. d. Reconciled to actual processing output. 133. Generalized audit software is a computer-assisted audit technique. It is one of the widely used technique for auditing computer application systems. Generalized audit software is most often used to a. Verify computer processing. b. Process data fields under the control of the operation manager. c. Independently analyze data files. d. Both a and b. 134. From an audit viewpoint, which of the following represents a potential disadvantage associated with the widespread use of microcomputers? a. Their portability. b. Their ease of access by novice users. c. Their easily developed programs using spreadsheets which do not have to be documented. d. All of the above. 135. Which of the following functions would have the least effect on an audit if it was not properly segregated? a. The systems analyst and the programmer functions. b. The computer operator and programmer functions. c. The computer operator and the user functions. d. The applications programmer and the systems programmer. 136. To obtain evidence that user identification and password control procedures are functioning as designed, an auditor would most likely a. Attempt to sign on to the system using invalid user identifications and passwords. b. Write a computer program that simulates the logic of the client’s access control software. c. Extract a random sample of processed transactions and ensure that the transactions were appropriately authorized. Examine statements signed by employees stating that they have not divulged their user identifications and passwords to any other person. 137. In considering a client's internal control structure in a computer environment, the auditor will encounter general controls and application controls. Which of the following is an application control? a. Organization charts. b. Hash total. c. Systems flowcharts. d. Control over program changes 138. Auditing by testing the input and output of a computer system-i.e., auditing "around" the computer--instead of the computer software itself will a. Not detect program errors that do not appear in the output sampled. b. Detect all program errors, regardless of the nature of the output. c. Provide the auditor with the same type of evidence. d. Not provide the auditor with confidence in the results of the auditing procedures. 139. Smith Corporation has numerous customers. A customer file is kept on disk. Each customer file contains the name, address, credit limit, and account balance. The auditor wishes to test this file to determine whether credit limits are being exceeded. The best procedure for the auditor to follow would be to a. Develop test data that would cause some account balances to exceed the credit limit and determine if the system properly detects such situations. b. Develop a program to compare credit limits with account balances and print out the details of any account with a balance exceeding its credit limit. c. Request a printout of all account balances so they can be manually checked against the credit limits. d. Request a printout of a sample of account balances so they can be individually checked against the credit limits. 140. Which of the following methods of testing application controls utilizes software prepared by the auditors and applied to the client's data? a. Parallel simulation. b. Integrated test facility. c. Test data. d. Exception report tests. 141. The test–data method is used by auditors to test the a. Accuracy of input data. b. c. d. Validity of the output. Procedures contained within the program. Normalcy of distribution of test data. 142. Which of the following is true of generalized audit software? a. They can be used only in auditing on-line computer systems. b. They can be used on any computer without modification. c. They each have their own characteristics, which the auditor must carefully consider before using in a given audit situation. d. They enable the auditor to perform all manual compliance test procedures less expensively. 143. Assume that an auditor estimated that 10,000 checks were issued during the accounting period. If an application control that performs a limit check for each check request is to be subjected to the auditor's test–data approach, the sample should include: a. Approximately 1,000 test items. b. A number of test items determined by the auditor to be sufficient under the circumstances. c. A number of test items determined by the auditor's reference to the appropriate sampling tables. d. One transaction. 144. PC DOS, MS DOS, and AppleDOS are examples of a. Application software. b. Generalized audit software. c. Database management systems. d. Operating software. 145. Which of the following is not an example of a computerassisted audit technique? a. Integrated test data. b. Audit modules. c. Disk operating systems. d. Audit hooks. 146. Which of the following statements most likely represents a disadvantage for an entity that maintains computer data files rather than manual files? a. It's usually more difficult to detect transposition errors. b. Transactions are usually authorized before they are executed and recorded. c. It's usually easier for unauthorized persons to access and alter the files. d. Random error is more common when similar transactions are processed in different ways. 147. Which of the following statements best describes a weakness often associated with computers? a. Computer equipment is more subject to systems error than manual processing is subject to human error. b. Computer equipment processes and records similar transactions in a similar manner. c. Control activities for detecting invalid and unusual transactions are less effective than manual control activities. d. Functions that would normally be separated in a manual system are combined in a computer system. 148. Accounting functions that are normally considered incompatible in a manual system are often combined by computer software. This necessitates an application control that prevents unapproved a. Access to the computer library. b. Revisions to existing software. c. Usage of software. d. Testing of modified software. 149. When software or files can be accessed from on-line servers, users should be required to enter a. A parity check. b. A personal identification code. c. d. A self-diagnosis test. An echo check. 150. An auditor's consideration of a company's computer control activities has disclosed the following four circumstances. Indicate which circumstance constitutes a significant deficiency in internal control. a. Computer operators do not have access to the complete software support documentation. b. Computer operators are closely supervised by programmers. c. Programmers are not authorized to operate computers. d. Only one generation of backup files is stored in an off-premises location. 151. In a computer system, hardware controls are designed to a. Arrange data in a logical sequence for processing. b. Correct errors in software. c. Monitor and detect errors in source documents. d. Detect and control errors arising from use of equipment. 152. In the weekly computer run to prepare payroll checks, a check was printed for an employee who had been terminated the previous week. Which of the following controls, if properly utilized, would have been most effective in preventing the error or ensuring its prompt detection? a. A control total for hours worked, prepared from time cards collected by the timekeeping department. b. Requiring the treasurer's office to account for the number of the pre-numbered checks issued to the CBIS department for the processing of the payroll c. Use of a check digit for employee numbers d. Use of a header label for the payroll input sheet 153. An auditor is preparing test data for use in the audit of a computer based accounts receivable application. Which of the following items would be appropriate to include as an item in the test data? a. A transaction record which contains an incorrect master file control total b. A master file record which contains an invalid customer identification number c. A master file record which contains an incorrect master file control total d. A transaction record which contains an invalid customer identification number. 154. Unauthorized alteration of on-line records can be prevented by employing: a. Key verification b. Computer sequence checks c. Computer matching d. Data base access controls 155. In auditing through a computer, the test data method is used by auditors to test the a. Accuracy of input data b. Validity of the output c. Procedures contained within the program d. Normalcy of distribution of test data. 156. In the preliminary survey the auditor learns that a department has several microcomputers. Which of the following is usually true and should be considered in planning the audit? a. Microcomputers, though small, are capable of processing financial information, and physical security is a control concern b. Microcomputers are limited to applications such as worksheet generation and do not present a significant audit risk c. Microcomputers are generally under the control of the data processing department and use the same control features d. Microcomputers are too small to contain any built-in control features. Therefore, other controls must be relied upon. 157. The primary reason for internal auditing's involvement in the development of new computer-based sysstems is to: a. Plan post-implementation reviews b. Promote adequate controls c. Train auditors in CBIS techniques d. Reduce overall audit effort. 158. Which of the following is an advantage of generalized computer audit packages? a. They are all written in one identical computer language b. They can be used for audits of clients that use differing CBIS equipment and file formats c. They have reduced the need for the auditor to study input controls for CBIS related procedures d. Their use can be substituted for a relatively large part of the required control testing 159. Processing simulated file data provides the auditor with information about the reliability of controls from evidence that exists in simulated files. One of the techniques involved in this approach makes use of a. Controlled reprocessing b. Program code checking c. Printout reviews d. Integrated test facility 160. Which of the following statements most likely represents a disadvantage for an entity that keeps microcomputer-prepared data files rather than manually prepared files? a. It is usually more difficult to detect transposition errors b. Transactions are usually authorized before they are executed and recorded c. It is usually easier for unauthorized persons to access and alter the files d. Random error associated with processing similar transactions in different ways is usually greater 161. The possibility of losing a large amount of information stored in computer files most likely would be reduced by the use of a. Back-up files b. Check digits c. Completeness tests d. Conversion verification 162. An integrated test facility (ITF) would be appropriate when the auditor needs to a. Trace a complex logic path through an application system b. Verify processing accuracy concurrently with processing c. Monitor transactions in an application system continuously d. Verify load module integrity for production programs 163. Where computer processing is used in significant accounting applications, internal accounting control procedures may be defined by classifying control procedures into two types: general and a. Administrative b. Specific c. Application d. Authorization 164. The increased presence of the microcomputer in the workplace has resulted in an increasing number of persons having access to the computer. A control that is often used to prevent unauthorized access to sensitive programs is: a. Backup copies of the diskettes b. Passwords for each of the users c. Disaster-recovery procedures d. Record counts of the number of input transactions in a batch being processed 165. Checklists, systems development methodology, and staff hiring are examples of what type of controls? a. Detective b. c. d. Preventive Subjective Corrective 166. When an on-line, real-time (OLRT) computer-based processing system is in use, internal control can be strengthened by a. Providing for the separation of duties between keypunching and error listing operations b. Attaching plastic file protection rings to reels of magnetic tape before new data can be entered on the file c. Making a validity check of an identification number before a user can obtain access to the computer files d. Preparing batch totals to provide assurance that file updates are made for the entire input 167. When auditing "around" the computer, the independent auditor focuses solely upon the source documents and a. Test data b. CBIS processing c. Control techniques d. CBIS output 168. One of the features that distinguishes computer processing from manual processing is a. Computer processing virtually eliminates the occurrence of computational error normally associated with manual processing b. Errors or fraud in computer processing will be detected soon after their occurrences c. The potential for systematic error is ordinarily greater in manual processing than in computerized processing d. Most computer systems are designed so that transaction trails useful for audit purposes do not exist 169. Given the increasing use of microcomputers as a means for accessing data bases, along with on-line real-time processing, companies face a serious challenge relating to data security. Which of the following is not an appropriate means for meeting this challenge? a. Institute a policy of strict identification and password controls housed in the computer software that permit only specified individuals to access the computer files and perform a given function. b. Limit terminals to perform only certain transactions. c. Program software to produce a log of transactions showing date, time, type of transaction, and operator. d. Prohibit the networking of microcomputers and do not permit users to access centralized data bases. 170. What type of computer-based system is characterized by data that are assembled from more than one location and records that are updated immediately? a. Microcomputer system b. Minicomputer system c. Batch processing system d. Online real-time system 171. Company A has recently converted its manual payroll to a computer-based system. Under the old system, employees who had resigned or been terminated were occasionally kept on the payroll and their checks were claimed and cashed by other employees, in collusion with shop foremen. The controller is concerned that this practice not be allowed to continue under the new system. The best control for preventing this form of "payroll padding" would be to a. Conduct exit interviews with all employees leaving the company, regardless of reason. b. Require foremen to obtain a signed receipt from each employee claiming a payroll check. c. Require the human resources department to authorize all hires and terminations, and to forward a current computerized list of active employee numbers to payroll prior to processing. Program the computer to reject inactive employee numbers. d. Install time clocks for use by all hourly employees. 172. Compared to a manual system, a CBIS generally 1) Reduces segregation of duties 2) Increases segregation of duties 3) Decreases manual inspection of processing results 4) Increases manual inspection of processing results. a. 1 and 3 b. 1 and 4 c. 2 and 3 d. 2 and 4 173. One of the major problems in a CBIS is that incompatible functions may be performed by the same individual. One compensating control for this is the use of a. Echo checks b. A self-checking digit system c. Computer generated hash totals d. A computer log 174. Which of the following processing controls would be most effective in assisting a store manager to ascertain whether the payroll transaction data were processed in their entirety? a. Payroll file header record b. Transaction identification codes c. Processing control totals d. Programmed exception reporting 175. An organizational control over CBIS operations is a. Run-to-run balancing of control totals b. Check digit verification of unique identifiers c. Separation of operating and programming functions d. Maintenance of output distribution logs 176. Which of the following methods of testing application controls utilizes a generalized audit software package prepared by the auditors? a. Parallel simulation b. Integrated testing facility approach c. Test data approach d. Exception report tests 177. An unauthorized employee took computer printouts from output bins accessible to all employees. A control which would have prevented this occurrence is a. A storage/retention control b. A spooler file control c. An output review control d. A report distribution control 178. Which of the following is a disadvantage of the integrated test facility approach? a. In establishing fictitious entities, the auditor may be compromising audit independence. b. Removing the fictitious transactions from the system is somewhat difficult and, if not done carefully, may contaminate the client's files. c. ITF is simply an automated version of auditing "around" the computer. d. The auditor may not always have a current copy of the authorized version of the client's program. 179. Totals of amounts in computer-record data fields which are not usually added for other purposes but are used only for data processing control purposes are called a. Record totals b. Hash totals c. Processing data totals d. Field totals 180. A hash total of employee numbers is part of the input to a payroll master file update program. The program compares the hash total to the total computed for transactions applied to the master file. The purpose of this procedure is to: a. Verify that employee numbers are valid b. Verify that only authorized employees are paid c. Detect errors in payroll calculations d. Detect the omission of transaction processing 181. Matthews Corp. has changed from a system of recording time worked on clock cards to a computerized payroll system in which employees record time in and out with magnetic cards. The CBIS automatically updates all payroll records. Because of this change a. A generalized computer audit program must be used b. Part of the audit trail is altered c. The potential for payroll related fraud is diminished d. Transactions must be processed in batches 182. Generalized audit software is of primary interest to the auditor in terms of its capability to a. Access information stored on computer files b. Select a sample of items for testing c. Evaluate sample test results d. Test the accuracy of the client's calculations 183. Accounts payable program posted a payable to a vendor not included in the on-line vendor master file. A control which would prevent this error is a a. Validity check b. Range check c. Reasonableness test d. Parity check 184. In a computerized sales processing system, which of the following controls is most effective in preventing sales invoice pricing errors? a. Sales invoices are reviewed by the product managers before being mailed to customers b. Current sales prices are stored in the computer, and, as stock numbers are entered from sales orders, the computer automatically prices the orders c. Sales prices, as well as product numbers, are entered as sales orders are entered at remote terminal locations d. Sales prices are reviewed and updated on a quarterly basis 185. Which of the following is likely to be of least importance to an auditor in reviewing the internal control in a company with a CBIS? a. The segregation of duties within the data processing center. b. The control over source documents c. The documentation maintained for accounting applications. d. The cost/benefit ratio of data processing operations 186. For the accounting system of Acme Company, the amounts of cash disbursements entered into an CBIS terminal are transmitted to the computer that immediately transmits the amounts back to the terminal for display on the terminal screen. This display enables the operator to a. Establish the validity of the account number b. Verify the amount was entered accurately c. Verify the authorization of the disbursement d. Prevent the overpayment of the account 187. Which of the following audit techniques most likely would provide an auditor with the most assurance about the effectiveness of the operation of an internal control procedure? a. Inquiry of client personnel b. Recomputation of account balance amounts c. Observation of client personnel d. Confirmation with outside parties 188. Adequate technical training and proficiency as an auditor encompasses an ability to understand a CBIS sufficiently to identify and evaluate a. The processing and imparting of information b. Essential accounting control features c. All accounting control features d. The degree to which programming conforms with application of generally accepted accounting principles. 189. Which of the following is not a major reason why an accounting audit trail should be maintained for a computer system? a. Query answering b. Deterrent to fraud c. Monitoring purposes d. Analytical review 190. Adequate control over access to data processing is required to a. Prevent improper use or manipulation of data files and programs b. Ensure that only console operators have access to program documentation c. Minimize the need for backup data files d. Ensure that hardware controls are operating effectively and as designed by the computer manufacturer 191. When testing a computerized accounting system, which of the following is not true of the test data approach? a. The test data need consist of only those valid and invalid conditions in which the auditor is interested b. Only one transaction of each type need be tested c. Test data are processed by the client's computer programs under the auditor's control d. The test data must consist of all possible valid and invalid conditions 192. In studying a client's internal controls, an auditor must be able to distinguish between prevention controls and detection controls. Of the following data processing controls, which is the best detection control? a. Use of data encryption techniques b. Review of machine utilization logs c. Policy requiring password security d. Backup and recovery procedure 193. Which of the following procedures is an example of auditing "around" the computer? a. The auditor traces adding machine tapes of sales order batch totals to a computer printout of the sales journal b. The auditor develops a set of hypothetical sales transactions and, using the client's computer program, enters the transactions into the system and observes the processing flow c. The auditor enters hypothetical transactions into the client's processing system during client processing of live" data d. The auditor observes client personnel as they process the biweekly payroll. The auditor is primarily concerned with computer rejection of data that fails to meet reasonableness limits 194. Auditing by testing the input and output of a computer-based system instead of the computer program itself will a. Not detect program errors which do not show up in the output sampled b. Detect all program errors, regardless of the nature of the output c. Provide the auditor with the same type of evidence d. Not provide the auditor with confidence in the results of the auditing procedures 195. Which of the following is an acknowledged risk of using test data when auditing CBIS records? a. The test data may not include all possible types of transactions b. The computer may not process a simulated transaction in the same way it would an identical actual transaction c. The method cannot be used with simulated master records d. Test data may be useful in verifying the correctness of account balances, but not in determining the presence of processing controls 196. When the auditor encounters sophisticated computer-based systems, he or she may need to modify the audit approach. Of the following conditions, which one is not a valid reason for modifying the audit approach? a. More advanced computer systems produce less documentation, thus reducing the visibility of the audit trail b. In complex comuter-based systems, computer verification of data at the point of input replaces the manual verification found in less sophisticated data processing systems c. Integrated data processing has replaced the more traditional separation of duties that existed in manual and batch processing systems. d. Real-time processing of transactions has enabled the auditor to concentrate less on the completeness assertion 197. If a control total were to be computed on each of the following data items, which would best be identified as a hash total for a payroll CBIS application? a. Net pay b. Department numbers c. Hours worked d. Total debits and total credits 198. In a distributed data base (DDB) environment, control tests for access control administration can be designed which focus on a. Reconciliation of batch control totals b. Examination of logged activity c. Prohibition of random access d. Analysis of system generated core dumps 199. A control to verify that the dollar amounts for all debits and credits for incoming transactions are posted to a receivables master file is the: a. Generation number check b. Master reference check c. Hash total d. Control total 200. The program flowcharting symbol representing a decision is a a. Triangle b. Circle c. Rectangle d. Diamond 201. An update program for bank account balances calculates check digits for account numbers. This is an example of a. An input control b. A file management control c. Access control d. An output control 202. CBIS controls are frequently classified as to general controls and application controls. Which of the following is an example of an application control? a. Programmers may access the computer only for testing and "debugging" programs b. All program changes must be fully documented and approved by the information systems manager and the user department authorizing the change c. A separate data control group is responsible for distributing output, and also compares input and output on a test basis d. In processing sales orders, the computer compares customer and product numbers with internally stored lists 203. After a preliminary phase of the review of a client's CBIS controls, an auditor may decide not to perform further tests related to the control procedures within the CBIS portion of the client's internal control system. Which of the following would not be a valid reason for choosing to omit further testing? a. The auditor wishes to further reduce assessed risk b. The controls duplicate operative controls existing elsewhere in the system c. d. There appear to be major weaknesses that would preclude reliance on the stated procedures The time and dollar costs of testing exceed the time and dollar savings in substantive testing if the controls are tested for compliance 204. For good internal control over computer program changes, a policy should be established requiring that a. The programmer designing the change adequately test the revised program b. All program changes be supervised by the CBIS control group c. Superseded portions of programs be deleted from the program run manual to avoid confusion d. All proposed changes be approved in writing by a responsible individual. 205. Which of the following is not a technique for testing data processing controls? a. The auditor develops a set of payroll test data that contain numerous errors. The auditor plans to enter these transactions into the client's system and observe whether the computer detects and properly responds to the error conditions b. The auditor utilizes the computer to randomly select customer accounts for confirmation c. The auditor creates a set of fictitious custom accounts and introduces hypothetical sales transactions, as well as sales returns and allowances, simultaneously with the client's live data processing d. At the auditor's request, the client has modified its payroll processing program so as to separately record any weekly payroll entry consisting of 60 hours or more. These separately recorded ("marked") entries are locked into the system and are available only to the auditor 206. Which of the following would lessen internal control in a CBIS? a. The computer librarian maintains custody of computer program instructions and detailed listings b. Computer operators have access to operator instructions and detailed program listings c. The control group is solely responsible for the distribution of all computer output d. Computer programmers write and debug programs which perform routines designed by the systems analyst 207. Access control in an on-line CBIS can best be provided in most circumstances by a. An adequate librarianship function controlling access to files b. A label affixed to the outside of a file medium holder that identifies the contents c. Batch processing of all input through a centralized, well-guarded facility d. User and terminal identification controls, such as passwords 208. While entering data into a cash receipts transaction file, an employee transposed two numbers in a customer code. Which of the following controls could prevent input of this type of error? a. Sequence check b. Record check c. Self-checking digit d. Field-size check 209. What is the computer process called when data processing is performed concurrently with a particular activity and the results are available soon enough to influence the particular course of action being taken or the decision being made? a. Batch processing b. Real time processing c. Integrated data processing d. Random access processing 210. Reconciling processing control totals is an example of a. An input control b. An output control c. A processing control d. A file management control 211. Disadvantage of auditing around the computer is that it a. Permits no assessment of actual processing b. Requires highly skilled auditors c. Demands intensive use of machine resources d. Interacts actively with auditee applications 212. The completeness of computer-generated sales figures can be tested by comparing the number of items listed on the daily sales report with the number of items billed on the actual invoices. This process uses a. Check digits b. Control totals c. Validity tests d. Process tracing data 213. Which of the following controls would be most efficient in reducing common data input errors? a. Keystroke verification b. A set of well-designed edit checks c. Balancing and reconciliation d. Batch totals 214. On-line real-time systems and electronic data interchange systems have the advantages of providing more timely information and reducing the quantity of documents associated with less automated systems. The advantages, however, may create some problems for the auditor. Which of the following characteristics of these systems does not create an audit problem? a. The lack of traditional documentation of transactions creates a need for greater attention to programmed controls at the point of transaction input b. Hard copy may not be retained by the client for long periods of time, thereby necessitating more frequent visits by the auditor c. Control testing may be more difficult given the increased vulnerability of the client's files to destruction during the testing process d. Consistent on-line processing of recurring data increases the incidence of errors 215. Creating simulated transactions that are processed through a system to generate results that are compared with predetermined results, is an auditing procedure referred to as a. Desk checking b. Use of test data c. Completing outstanding jobs d. Parallel simulation 216. To obtain evidential matter about control risk, an auditor ordinarily selects tests from a variety of techniques, including a. Analysis b. Confirmations c. Reprocessing d. Comparison 217. A major exposure associated with the rapidly expanding use of microcomputers is the absence of: a. Adequate size of main memory and disk storage b. Compatible operating systems c. Formalized procedures for purchase justification d. Physical, data file, and program security 218. To ensure that goods received are the same as those shown on the purchase invoice, a computerized system should: a. Match selected fields of the purchase invoice to goods received b. Maintain control totals of inventory value c. Calculate batch totals for each input d. Use check digits in account numbers 219. Errors in data processed in a batch computer system may not be detected immediately because a. Transaction trails in a batch system are available only for a limited period of time b. There are time delays in processing transactions in a batch system c. Errors in some transactions cause rejection of other transactions in the batch d. Random errors are more likely in a batch system than in an on-line system 220. Which of the following is a computer test made to ascertain whether a given characteristic belongs to the group? a. Parity check b. Validity check c. Echo check d. Limit check. ...

Which of the following is the least likely characteristic of personal computers?

User generated content is uploaded by users for the purposes of learning and should be used following Studypool's honor code & terms of service.

Which of the following is the least likely characteristic of personal computers?

Studypool

4.7

Which of the following is the least likely characteristic of personal computers?

Trustpilot

4.5

Which of the following is the least likely characteristic of personal computers?

Sitejabber

4.4

What are the characteristics of a personal computer?

Speed. A computer works with much higher speed and accuracy compared to humans while performing mathematical calculations. ... .
Accuracy. Computers perform calculations with 100% accuracy. ... .
Diligence. A computer can perform millions of tasks or calculations with the same consistency and accuracy. ... .
Versatility. ... .
Reliability. ... .
Memory..

Which is not a type of personal computer?

Which of the following is not a type of personal computer? Mainframe computer is more expensive and big computer. More than one user will be processing data at a time. It concurrently accomplishes thousands of data processing.

Which one of the following represents a lack of internal control in a computer based information system?

Which one of the following represents a lack of internal control in a computer-based system? Programmers have access to change programs and data files when an error is detected.

Which of the following statements most likely represents a disadvantage for an entity that maintains computer data files rather than manual files?

Which of the following is most likely a disadvantage for an entity that keeps data files prepared by personal computers rather than manually prepared files? It is usually easier for unauthorized persons to access and alter the files.